You are on page 1of 84

CONSTITUTIONAL LAW

History of Due Process and Fundamental Rights Today s fundamental rights or substantive due process doctrine is the jurisprudential successor to the Lochner era s economic substantive due process 1. Lochner focus was on economic rights e.g., contract and property 2. Today, the focus is on non-economic or personal rights e.g., privacy and autonomy 3. Look for common threads Three historical phases 1. The first century (1787 to 1890s): very little attention paid to individual rights 2. The Lochner era (1890s to 1930s): focus on economic and property rights 3. The fundamental rights era (1930s to present): successor to economic due process
y

Must rights be grounded in text the Constitution or the Bill of Rights?

Slaughterhouse Cases (1873) y Upheld LA law granting a monopoly to operate slaughterhouses to keep the noxious and offensive animals out of the city. Excluded butchers claimed 13th and 14th Amendment violations. The court cites that the amendments were for restoring order, and to give protection to the freed slaves. Without them, they would be in just as bad a situation as slaves. That fact does not limit the benefits of the amendments to slaves, but the purpose of them needs to be taken into account. Also, it points out that the law has not deprived or taken away anything away from the appellants.
y

Three things are at issue: 1. 14th Amendment Privileges & immunities Clause Protects rights of US citizens. But there are not many rights as citizens, and strangles the 14th in this point. 2. Due process No way anything here that it has anything to do with due process violation, essentially stating the 14th has nothing to do with due process 3. Equal protection Court points out that equal protection will apply to anyone other than freed slaves. This case is still good law as to Privileges & Immunities, even if it doesn t have much impact. Due process and equal protection are no longer held through this case. This would eventually prompt the Court to look elsewhere for due process and equal protection. Justice Miller cannot see that the 14th has changed things to the extent in which it has. P301 he points out that the 14th couldn t have meant the changes it actually states.

y y

Changes slowly began afterward, and Lochner is the key case that brings the 14th into play.

Page | 1

Justice Field: The State may prescribe such regulations for every pursuit and calling of life as will promote the public health, secure the good order and advance the general prosperity of society, but once prescribed, the pursuit or calling must be free to be followed by every citizen who is within the conditions designated . The Rise of Substantive Due Process The due process clauses of the 5th and 14th Amendments contain a substantive component. The key problems with these issues are that rights based on this are NOT enumerated in the Constitution.
y

Framers diffused the power of government to three branches and gave substantive rights in this manner. Having a gov t of enumerated and limited power also protected rights. Not all Framers felt the Constitution gave enough limitation, and the Bill of Rights was the result. Federalists Madison and Hamilton thought the Bill of Rights could be dangerous based on the thinking that expressing some would exclude everything else, that s why the Constitution didn t have anything specific (expressio unius) Madison ended up drafting the first Bill of Rights. The problem with the Bill of Rights is that it binds only the Federal gov t Barron. Barron alleged an unconstitutional taking w/o fair compensation. The case was dismissed because state government did not cover fair compensation. th y The 14 ended up tying states to the same limitations as the federal government.

14th Amendment Tells us that all persons born or naturalized in the US are citizens and the state where they reside. Being subject to the jurisdiction (excludes diplomats, etc.) is also relevant to citizenship. The purpose of the first sentence of the Amendment was to overturn Dred Scott. It then points out that states cannot abridge privileges or immunities or deprive any person of due process of law or equal protection of the law. Privileges & Immunities, Due Process, Equal Protection are all covered in 14th. The intent of the 14th framers was that the first 8 amendments would be binding on the states. Prior to the 14th, if individual rights were being violated, the constitution presumed you would look to the state for protection. Post 14th, this was now a matter of federal law and away from state police power. The Reconstruction Amendments had the southern states in mind specifically. It basically stated that the states can t be relied upon for individual protections. This was a huge change from the original founding. Liberty interests and fundamental rights are different Fundamental rights are a subset of liberty interests. All fundamental rights are liberty interests, but not all liberty interests are fundamental rights. Example The right to use contraception how is it protected when it is not specifically mentioned. And why aren t other things protected? Why can t you terminate your own life if you are terminally

y y

Page | 2

ill w/ physician assistance? The indictment against substantive due process is that the Courts are simply making it up as they go along. 1. Post-Slaughterhouse, where is the rights pendulum leaning? 2. Things began to change a. Mugler v. Kansas (1887): saying law enacted pursuant to police power doesn t make it so; courts will inquire whether law has real or substantial relation to police power objects b. Allgeyer v. Louisiana (1897): right to contract wins over police power The Lochner era The Court discovered that it was a protected liberty interest to enter into a contract. As such, it deserves a special protection. But it isn t mentioned in the Constitution. This era eventually passed in the 1930 s when Commerce Power came into play. This carried over to 14th Amendment due process rights. The Court proclaimed it would no longer scrutinize due process issues, and that it rested upon state police power. They are going to presume social and economic legislation is constitutional, and will not challenge it unless no rational basis has been established. This made Court review extremely deferential.

Lochner v. New York (1905) y Key issue is that the 14th says nothing about the right to contract, but the case supports the idea that a liberty interest that should have constitutional protection.
y

The court sees this concern as arbitrary, and if the actual purpose is a health issue, is this the way to address it? Their approach of looking at how a policy is being addressed is that it s not for the Court to do, it s the Legislature. And even if it s denying that it s doing it, it really is. This case illustrates that the constitution protected the right to contract (at the time). Today, this is seen as the paradigm for judicial policymaking. Once the Court detaches itself from the Constitution, critics see that the Court is now lawmaking on its own. The question then becomes how far away from the Constitution are you going when establishing unenumerated rights? 14th Amendment right to make a contract in relation to his business is part of the [individual] liberty protected by the 14th Amendment. Is this a text-based right? 1. 2.
y

No reasonable ground for interfering with right of bakers to determine hours of labor No reasonable foundation for law as means to protect public health or health of bakers The Court s Key Points: 1. If the state can regulate bakers it can regulate any profession 2. Statute is a mere meddlesome interference with the rights of the individual and is not saved from condemnation by the claim that [it is] passed in the exercise of the police power

Page | 3

3. Court denies that it is substituting its own judgment for that of legislature
y

Justice Holmes 1. Majority, acting through legislature, gets to make the laws 2. Constitution not meant to enact an economic theory of paternalism or laissez faire 3. Statute should be upheld unless it would infringe fundamental principles as they have been understood by the traditions of our people and our law.

End of an Era Nebbia v. New York (1934) y The beginning of the end of the Lochner era begins here. The Court upheld a price fixing law on milk. This is part of the state police power, and the state of NY is regulating prices in order to protect milk producers.
y

The Court sees the right to contract important (Lochner) but it is more important for the states to regulate prices for the good of the state. The tension is between individual rights versus rights of society; what is the balance? This applies to all individual rights issues and the balance between the two will always be at issue. The power to control the individual is fundamental to societal rights. Ex. You have right to do as you please with your property, but not to the extent that it will devalue the property of your neighbors (society). The Court is moving to the rationality standard with Nebbia in evaluating due process issues. Previously, the Court had found that a variety of price regulation laws violated substantive due process Justice Roberts signaled the end r State s right to regulate in the public interest is equally fundamental to the individual right to freedom of contract r Due process means only that law not be unreasonable, arbitrary, or capricious and that means have real and substantial relation to the end state seeks to achieve Textual argument: Due process clause doesn t mention sales, prices, business, contracts Competence argument: Courts are without authority either to declare [economic] policy, or, when it is declared to override it. See also West Coast Hotel v. Parrish (1937)

The Birth of Modern Substantive Due Process United States v. Carolene Products (1938) r The Fill Milk Act banned the sale of fake milk. The Act came about from concerns of the dairy industry affecting their market, and as a result Congress created the act. Carolene argued the Act was a violation of the freedom to contract.
r

Case is significant because it uses the approach that ordinary economic social legislation is entitled to a presumption of Constitutionality and must be upheld if it s supported by any rational basis.

Page | 4

If the rational basis is challenged, inquiry is limited to facts known or reasonably assume them and as a result play the what if game. Even the reasons Congress relied on to make the legislation do not have to be taken into consideration. This case shows that the Court is leaving the substantive due process issue and fully departing from the Lochner era. This puts the ball back to the political process court to determine what is wanted or not since the Court will not make decisions on the merit of legislative acts. The Most Famous Footnote Carolene Products Footnote 4 1. Points out three categories of regulations that will be more scrutinized: a. Legislation that on its face is within specific prohibition such as found in Bill of Rights (If legislation facially impinges on a fundamental right) b. Legislation that restricts or impairs the political processes c. Legislation directed at racial, national, or religious minorities; prejudice against discrete and insular minorities may curtail operation of political processes. i. Discrete means: spate and distinct. ii. Insular means: isolated. 2. The footnote establishes certain categories that the Court would review. Today s fundamental rights doctrine came from this. Griswold is a key fundamental rights issue, and substantive right of couples to use contraception. And how does this carry over to unmarried people do they have lesser rights than a married person? 3. What gets special attention? a. Are courts out of substantive due process business? What is not pointed out is how closely they will be looked at. The Court isn t really completely out of the substantive due process business; it has simply established some guidelines in which to approach it.

Modern Substantive Due Process Williamson v. Lee Optical (1955) r Stands for proposition that ordinary social or economic legislation will not be scrutinized r [T]he law need not be in every respect logically consistent with its aims to be constitutional. r Cannot say the law has no rational relation to a legitimate end r Here there is a presumption of constitutionality because there is no fundamental right or state law causing impairment. r It will only have a rational basis review. In this case, the will have to prove that the law doesn t have a legitimate state interest. It can be as simple as saving money so it will be hard to beat. r Also, the means selected are not rationally related to a legitimate end. Carolene any state of facts known or assumed affords support for the law. Basically, that means the court will play what if.

Page | 5

Regulations face one of the following types of review: 1. Rational Basis Regulations are very likely to survive rational review. 2. Strict Scrutiny When a fundamental right is at stake, and the court looks at the history and tradition of protecting the right. Also, it will look at whether it is implicit in ordered liberty. r For exam mention both under strict scrutiny question. The question to ask is if the law looks like it is aimed at a right, or if it s incidental. r If it s the first, substantial impairment, if the latter, no substantial impairment. r With a substantial impairment, burden shifts to gov t to show compelling interest and means are narrowly tailored. Namely, the least restrictive means available. If the court can find something less restrictive, it s out. r Means testing more often fails than ends testing

Ferguson v. Skrupa (1963) r Lochner [has] long since been discarded. r Competence: We have returned to the original constitutional proposition that courts do not substitute their social and economic beliefs for the judgment of the judgment of legislative bodies, who are elected to pass laws. r Justice Harlan: law bears a rational relation to a constitutionally permissible objective. Post Lochner and Carolene Products 1. Courts will no longer closely scrutinize most social and economic legislation 2. Focus is on fundamental rights; three questions a. What rights are fundamental? b. Where do we locate these rights? c. Who gets to identify and decide the scope of these rights?

Fundamental Rights General Principles Certain rights what are sometimes called fundamental rights are so important that courts will closely scrutinize any attempt by the government to regulate their exercise. Fundamental rights may be r Textual (found in Constitution or Bill of Rights) or r Non-textual Our concern, at this point, is with non-textual fundamental rights General Rules 1. Any regulation that substantially impairs exercise of a fundamental right will be strictly scrutinized by the courts, and will be upheld only if necessary or narrowly tailored (least restrictive means) to meet a compelling government interest.

Page | 6

2. If no fundamental right is involved, regulation will be upheld if it bears a rational relationship to a legitimate state interest

DUE PROCESS: BASIC ANAYLITICAL STRUCTURE


Used for determining whether a due process violation exists within a regulation. 1. Is it a fundamental right? r If no, rational basis test applied r If yes, go to Question 2 2. If yes, is the right being substantially impaired? r If no, rational basis test applied r If yes, examine government interest 3. If yes, is there a compelling interest to justify the impairment? r If no, then due process violation r If yes, then do means testing 4. If yes, are the means necessary or narrowly tailored to achieve that interest? r If no, then due process violation r If yes, then no due process violation

Note that there are some exceptions to the above analysis, such as for abortion cases 1. Fundamental right to abortion 2. Has government unduly burdened the right? What are courts really doing here? y Balancing the interests of the individual and the state o For fundamental rights, very careful balancing o For everything else, little or no real balancing

Non-Textual Fundamental Rights Are determined by whether: 1. The asserted right is implicit in the concept of ordered liberty? 2. The asserted right is deeply rooted in our nation s history or traditions?

Page | 7

Fundamental Rights Procreation & Contraception The Court has identified a right to privacy or autonomy that protects from government intrusion certain fundamental rights; one of these is right to procreate Probably most accurate to think of a category of procreative choice rights Includes both right to procreate and right not to procreate contraception and abortion

Procreation r Procreation cases illustrate early evolution of fundamental rights doctrine r Early (Lochner era) cases did not speak of fundamental rights; sounded like rational basis r Court struggled to articulate a coherent doctrine Skinner v. Oklahoma r We are dealing here with legislation which involves one of the basic civil rights of man. Marriage and procreation are fundamental to the very existence and survival of the race. r Strict scrutiny of classification Buck v. Bell (1927) Forced sterilization of mental defectives Court did not find fundamental right to procreate [T]he public welfare may call upon the best citizens for their lives. . It is better for all the world, if society can prevent those who are manifestly unfit from continuing their kind. . Three generations of imbeciles are enough.

Conception Griswold v. Connecticut (due process right of married couples) r Note the express disavowal of Lochner r Institutional competence: Court will not sit as a super-legislature to determine the wisdom, need, and propriety of laws that touch economic problems, business affairs, or social conditions. r What is a penumbra? How far do these penumbras extend? r Who decides? r How do we decide? r The heart of the analysis r Fundamental right at issue a relationship lying within the zone of privacy created by several fundamental constitutional guarantees. r What does Griswold mean? r Fundamental right to use contraceptives? r Fundamental right to marital intimacy? r Married couples have fundamental right to use contraceptives? r Justice Goldberg r Liberty embraces the right of marital privacy . r Based conclusion upon 9th Amendment; language and history show that Framers believed there were fundamental rights not enumerated that were protected from government infringement Eisenstadt v. Baird (equal protection right extends fundamental right to non-married persons) r Right is fundamental as to single persons, too

Page | 8

If the right of privacy means anything, it is the right of the individual, married or single, to be free from unwarranted governmental intrusion into matters so [fundamental] as the decision whether to bear or beget a child.

Fundamental Rights Marriage, Family, and Personal Autonomy The right to privacy or autonomy includes, among others, the rights to marry, live as a family, have custody, care, and control of your children, make certain medical decisions, and engage in certain consensual, intimate conduct. Not all of these rights have been declared fundamental, but they have been protected Marriage Loving v. Virginia (1967) r Marriage is one of the basic civil rights of man, fundamental to our very existence and survival. Zablocki v. Redhail (1978) r Is it a fundamental right? Does it matter how you describe the right? How might you describe the right so that it is not fundamental? r Court says [T]he fundamental character of the right to marry . r Court says that statute clearly does interfere directly and substantially with the right to marry. Some are absolutely prevented from marrying Others will be coerced into foregoing right to marry Parental Right to Child Custody Stanley v. Illinois (fundamental right) r Court noted that integrity of the family unit has been protected via equal protection, due process, and Ninth Amendment r [A]t the least, Stanley s interest in retaining custody of his children is substantial. Michael H v. Gerald D. r Custody limitation case; fathered a child adulterously w/ mother while married to another, no fundamental right to be recognized as the parent. r More importantly, FN6 states that one must articulate at the most specific level a tradition or history protecting or denying the proclaimed right can be identified (instead of the broad perspective) It is NOT a rule of law, did not receive a majority of votes. r [T]he legal issue reduces to whether the relationship between persons in the situation of Michael and Victoria has been treated as a protected family unit under the historic practices of our society Parents have fundamental right to care for and control upbringing of children Derives from Meyer v. Nebraska and Pierce v. Society of Sisters Meyer v. Nebraska (1923): 14th Amendment liberty includes: Rights to contract, engage in common occupations of life, and acquire useful knowledge

Page | 9

Rights to marry, establish a home, bring up children, and worship God Right to enjoy privileges essential to orderly pursuit of happiness [T]his liberty may not be interfered with, under the guise of protecting the public interest, by legislative action which is arbitrary or without reasonable relation to some purpose within the competency of the State to effect. Did not call these liberties fundamental

Pierce v. Society of Sisters (1925) Oregon law mandated attendance at public schools The Act unreasonably interferes with the liberty of parents to direct the upbringing and education of children State cannot abridge rights by legislation which has no reasonable relation to some purpose within the competency of the state.

Troxel v. Granville (plurality) (fundamental right) Parents rights to care, custody, and control of their children is the oldest of the fundamental liberty interests recognized . r Washington statute, as applied, unconstitutionally infringes on fundamental parental right r As a general rule, rights of parents to custody of their children, and to raise their children, are fundamental. See, e.g., Meyer; Skinner r Application of this general rule is less clear in cases where child is illegitimate (born out of wedlock) or is the child of an adulterous affair with a married woman
r

Right of intimate association to live together as family Moore v. City of East Cleveland (plurality) Reflects evolution of heightened scrutiny [F]reedom of personal choice in matters of marriage and family life is a protected due process liberty History and tradition [T]he institution of the family is deeply rooted in this Nation s history and traditions. At what level of generality is this history and tradition cast? Competent adults have a right to make certain decisions relating to health care Cruzan v. Director, Missouri Dept. of Health But cf. Jacobson v. Massachusetts (state has compelling interest in preventing spread of communicable diseases) Washington v. Glucksberg (1997) Is there a fundamental right? Court examines history and tradition and finds no protection for assisting in suicide; history and tradition not only prohibit that, but prohibit suicide as well Court has assumed, and strongly suggested, that the Due Process Clause protects the traditional right to refuse unwanted lifesaving medical treatment. Opinion says that grounding fundamental rights in history and tradition tends to rein in the subjective elements that are necessarily present in due process review.

Page | 10

Court also says this method eliminates need for complex balancing of interest in every case. Court has not recognized a fundamental right to commit suicide with physician assistance

Court has protected the right of consenting adults to participate in certain private, intimate conduct Bowers v. Hardwick (1986) Is there a fundamental right? How does Court describe the right at issue? Why does the Court describe it this way when the statute was not limited to same-sex conduct? Applied rational basis test Is the belief of a presumed majority regarding the morality or immorality of conduct enough, by itself, to legitimate a state interest? Is morality the ultimate rational basis trump card? Constitution does have the specific act listed, compared to Lawrence, looks at the act as a private, intimate act. Michael H is the other side of this with the FN being so specific.

Lawrence v. Texas r Note that Court has not declared a fundamental right to engage in this conduct r Nor has Court declared that homosexuality is a suspect classification requiring strict scrutiny r Scope of privacy right r Statute sought to control a personal relationship that whether or not entitled to formal recognition in the law, is within the liberty of persons to choose (emphasis added) r This should counsel against attempts by the State to define the meaning of the relationship or to set its boundaries absent injury to a person or abuse of an institution the law protects. (emphasis added) r Due process allows homosexual persons the right to make this choice. r Laws and traditions of the past half century are of most relevance here. r An emerging awareness that liberty gives substantial protection to adult persons in deciding how to conduct their private lives in matters pertaining to sex. r What interest does the state assert? r How does Court respond to this interest? r Court does not explicitly apply rational basis test r No discussion of whether statute rationally related to legitimate state interest r Court simply says there is no legitimate state interest to justify state intrusion into private life of the individual r Rational basis plus ?

Substantive due process question will posit a right not already declared to be fundamental - Tradition - General level - Rhetorical game, assessing both equally to arrive at an answer to whether the right in question is in fact fundamental

Page | 11

DUE PROCESS / FUNDAMENTAL RIGHTS: EXERCISE EXAMPLE


Abigail case terminally ill cancer patients who have exhausted all FDA treatments and are not able to participate in clinical trials, they have no options. The argument is that these patients have a fundamental right to Phase 1 FDA approved medications (they haven t been fully approved yet). Analyze: is this a fundamental right under the liberty. Exam pointer analysis, not a fundamental right, finished. BAD. Need to put into analysis other side of the analysis. 1. Is it a fundamental right in a behavior to preserve or lengthen their life? Yes (broad based) challenger has burden of proof a. Is there a fundamental right to experimental right to drugs not FDA approved? Specific No there is no history or tradition protecting it. b. Is it a fundamental right to make decisions about my own body? General c. Right to refuse treatment, right to accept treatment? d. Fundamental right to medical treatment that is necessary to preserve or prolong my life? 2. Assuming it is a fundamental right, is the right being substantially impaired? Challenger bears burden of proof a. Is the right being directly burdened or incidental burden? Was the gov t aiming directly at this right, or was it incidental in impairment and concerned about something entirely different? b. Argument: No substantial impairment because the gov t is protecting the integrity of the medical community and not trying to deprive people of new drug

3. If yes on both, then gov t has to show a compelling interest no direct indication given by the Court. Gov t bears burden of proof. Make sure to mention the burden shift. a. Government s argument could be keeping the public safe, keeping unsafe drugs off the street, preserving and protecting the drug approval process. b. Patient s argument could be that limiting the release of the drug won t undermine the process, preserving a life is more compelling than preserving the process. c. Compelling interests is weighing the rights of the individual versus the society s. 4. If compelling interest, means testing, are the means necessary and narrowly tailored. Gov t bears burden of proof. a. Are these the least restrictive means? Almost certainly, yes. b. As a challenger, you re in a strong position to win at this point while the government is looking for rational basis review.

Page | 12

Fundamental Rights Abortion The right to choose at least under some circumstances has been seen by the Court as a fundamental right. This is analyzed in a different way. When the right includes the right to choose abortion, the undue burden standard is used. Namely, does this law place an undue burden on the right? If the regulation has the purpose or effect of placing a substantial obstacle in the path of a woman who seeks to abort a non-viable fetus it is an undue burden. Viability is the crucial issue at hand. A regulation does not create an undue burden if it regulates the decision post viability. By definition, an undue burden is an issue only during viability. The government can regulate abortions for two of its interests: 1. Protecting maternal health 2. Protecting potential or prenatal life

Due process analysis is modified


The right is fundamental Is the right being unduly burdened? If yes, regulation is invalid If no, regulation is tested using rational basis test Critical point is viability Reasonable pre-viability regulations to protect maternal health do not unduly burden the right To protect its interest in prenatal or potential life, state may also regulate pre-viability to further its policy of encouraging childbirth, so long as the regulations do not unduly burden right Post-viability, government has compelling interest in protecting prenatal or potential life and may ban abortion completely (if there are exceptions for maternal life or health?) Government not required to fund non-therapeutic abortions or provide facilities where they are performed For minors, parental notification or consent may be required, as long as a judicial bypass procedure is in place State cannot require spousal notification or consent

Roe v. Wade This right of privacy, whether it be founded in the Fourteenth Amendment's concept of personal liberty and restrictions upon state action, as we feel it is, or, as the District Court determined, in the Ninth Amendment's reservation of rights to the people, is broad enough to encompass a woman's decision whether or not to terminate her pregnancy. Blackmun wrote the opinion for Roe, who was previously general counsel for the Mayo Clinic.

Page | 13

Government has two interests to protect in this case 1. Maternal health 2. Prenatal or potential life No Personhood Finding: Blackmun looks for the definition of person in the Constitution textually and found it was always used in a post-natal context. From that, he concludes that unborn fetuses are not persons. Had the Court saw a fetus as person, now there is a due process requirement for the fetus. When determining when life begins was not taken by the Court under an institutional competence reasoning if the medical field can t determine this, there s no way we could either. However, the Court has been able to define death, why can t they determine when life begins? Why is the determining when life begins constitutionally irrelevant? Because the rights under the 14th and 5th are for a person. By that, not being a person under the constitution means you don t have these rights. Roe s argument is that they should be able to have an abortion whenever they want. The court disagrees, looking at how over time the state interest of the potential life increases until they then becomes compelling. How can the state address the death of an unborn when it isn t a person? Looks at homicide as the taking of a life, the term person doesn t come into play. What does Roe adopt? A balancing approach and the trimester viability approach. 1. State can t regulate abortion prior to the end of the 1st trimester. 2. After that trimester, under an assumption that viability has occurred, the state s interest in protecting the fetus takes priority and can now regulate. Viability in legal terms is defined in Danforth case live with or without life support outside the womb.

Pre-viability no regulations Post-viability regulations ok

Roe is primarily discussed in the position of the doctor s rights.

Analysis
Is it a fundamental right? Yes Major premise: Only fundamental personal rights are protected under the constitutional right to privacy Minor premise: The right to privacy includes the right to decide whether to end a pregnancy Conclusion: The right to decide whether to end a pregnancy is a fundamental right Did Texas law substantially impair this right? The detriment that the State would impose upon the pregnant woman by denying this choice altogether is apparent. A variety of harms may follow: Specific and direct harm medically diagnosable

Page | 14

Psychological harm Mental and physical health may be taxed by child care

What are the state s interests? Protecting maternal health Protecting prenatal life The term person does not include the unborn. When does life begin? We need not resolve the difficult question of when life begins. Does it matter if the fetus is not a person? [W]e do not agree that, by adopting one theory of life, Texas may override the rights of the pregnant woman that are at stake. State can regulate when its interests become sufficiently compelling Maternal health: at end of first trimester Prenatal life: at viability. Problems? Viability exists if fetus can live outside womb with or without artificial life support. See Planned Parenthood v. Danforth (1976); Colautti v. Franklin (1979) After viability state may ban completely, (but only with exceptions for maternal life and health)

Evolving Standards Akron v. Akron Center for Reproductive Health (Akron I) (1983) (O Connor, J., dissenting) Undue burden standard should be applied throughout pregnancy, regardless of trimester If regulation creates undue burden, then state must articulate compelling interest and show that regulation is necessary or narrowly tailored If regulation does not create undue burden, then test is rational basis

Webster v. Reproductive Health Svcs. (1989) Preamble Preamble expressed a permissible value judgment by State that childbirth favored over abortion. We think the extent to which the preamble's language might be used to interpret other state statutes or regulations is something that only the courts of Missouri can definitively decide. Viability testing We must first determine the meaning of [the viability testing requirement] under Missouri law. Why must the Court decide the meaning of this statute, when only Missouri courts are competent to decide the meaning of the preamble? If the Court is not competent to decide one, is it competent to decide the other? Does one operate differently than the other? Viability testing does more To the extent that [the viability testing statute] regulates the method for determining viability, it undoubtedly does superimpose state regulation on the medical determination whether a particular fetus is viable.

Page | 15

Does a legislative finding that life begins at conception superimpose state regulation upon any medical determinations? Problem is not with viability testing it is with the Roe trimester framework, which has turned the law into a virtual Procrustean bed. Trimesters and viability are not mentioned in the Constitution Viability testing need not meet strict scrutiny Something like the rational basis test should be applied: The Missouri testing requirement here is reasonably designed to ensure that abortions are not performed where the fetus is viable - an end which all concede is legitimate - and that is sufficient to sustain its constitutionality.

Planned Parenthood v. Casey (1992) Use this case and standard on exam and bar exam. Historical note initial vote 5-4 overruled Roe. Kennedy flipped. The undue burden standard comes out of this. Not official because of no majority, but Stenberg makes it law. Strict scrutiny is usually used to test for fundamental rights. Without its usage, it s no longer a fundamental right on a strict basis. However, the Court has not specifically stated abortion to no longer be a fundamental right. Essentially: Part I (upholding the essential holding of Roe); Part II (right to abortion is grounded in 14th Amendment due process liberty); Part III (precedential force of Roe); Part V-C (striking spousal notification): O Connor, Kennedy, Souter, Blackmun, and Stevens Part IV (eliminating trimesters; undue burden standard): O Connor, Kennedy, and Souter Part I: reaffirmed essential holding of Roe First, right to have abortion pre-viability without undue interference from the state State cannot prohibit pre-viability abortion State cannot place substantial obstacle in way of right to choose pre-viability abortion Second, state can ban abortions post-viability, if there is an exception for maternal life and health Third, state has legitimate interests in protecting maternal health and fetal life throughout pregnancy Part II: right to abortion grounded in 14th Amendment due process liberty Neither the Bill of Rights nor the specific practices of States at the time of the adoption of the Fourteenth Amendment marks the outer limits of the substantive sphere of liberty which the Fourteenth Amendment protects. These matters are central to the liberty protected by the Fourteenth Amendment. Liberty is not limited to those in the Bill of Rights, meaning there must be unenumerated rights protected by the 14th due process. History and tradition is important, but not the end point. Part III: precedential force of Roe Considerations in determining precedential force None of these factors weighed in favor of overruling Roe Stare decisis standpoint, pointing out if the old rules are not longer in place, use of the ruling, credibility challenged, it should be overruled.

Page | 16

Without all this, and under Roe, no overruling.

Part IV: trimesters and undue burden standard Though no majority opinion here, a majority did agree that the critical issue is viability, so trimester framework was effectively abandoned Undue burden: a regulation which has the purpose or effect of placing a substantial obstacle in the way of a woman seeking to abort a non-viable fetus Means chosen by state to further interest in potential life must be intended to inform woman s free choice, not hinder it Law designed to further state interest in fetal life which imposes undue burden on decision pre-viability is unconstitutional Measure designed to encourage childbirth will be upheld if reasonably related to that goal, if no undue burden Regulations to protect maternal health valid if no undue burden Part IV summary The law creates an undue burden and is invalid if its purpose or effect is to place a substantial obstacle in the way of pre-viability abortion Throughout pregnancy state may further interest in potential life by ensuring choice is informed. Regulations must not create undue burden State may regulate to protect maternal health. Regulations with purpose or effect of presenting a substantial obstacle are an undue burden State may not prohibit abortion pre-viability State may prohibit abortion post-viability if exceptions for maternal life and health

The New Standard Stenberg v. Carhart (2000) Nebraska statute prohibited partial birth late-term abortion procedure Majority invalidated statute because No health exception as to the less commonly-used procedure By banning more commonly-used abortion procedure, it placed an undue burden on the abortion right Stenberg was the first majority decision to adopt the undue burden standard

Late-Term Abortion Gonzales v. Carhart (2007) Federal law banned intact D & E abortion procedure No health exception provided Conflicting medical evidence as to whether banning procedure would increase late-term abortion risk for certain women Court split 5-4; Justice Kennedy wrote for majority Ban did not create undue burden; rational basis standard applied No substantial obstacle to late-term, pre-viability abortions What about informed consent? [The] State has an interest in ensuring so grave a choice is well informed.

Page | 17

[T]his lack of information concerning the way in which the fetus will be killed is of legitimate concern to the state. Even if documented medical disagreement about health risks from ban, the ban survives Where Stenberg erred on side of woman s choice, Carhart errs on side of state interest in life Legislatures have wide discretion to make laws where there is medical and scientific uncertainty. Facial challenges to abortion laws inappropriate; proper means is as-applied

Post-Stenberg As of today; five votes to uphold basic abortion right and in favor of undue burden standard: Justices Stevens, Kennedy, Souter, Breyer, and Ginsburg Two votes to overrule Roe: Justices Scalia and Thomas Funding and Facilities

State funding and facilities Maher v. Roe (1977) State policy funding childbirth but not abortion imposes no restriction on access to abortion not already there Since non-funding does not impair a fundamental right, test is rational basis. State has a strong and legitimate interest in encouraging childbirth and funding scheme is a rational means of meeting state interest Poelker v. Doe (1977): state may provide public hospitals for childbirth and withhold services for nontherapeutic abortions Federal government is not required to fund non-therapeutic abortions Harris v. McRae (1980) Freedom to choose abortion does not mean constitutional entitlement to funding for abortion Rust v. Sullivan (1991) Federal regulations may prohibit federally funded clinics from providing counseling for nontherapeutic abortions No worse off than if Congress provided no funding for family planning services at all Webster v. Reproductive Health Svcs. (1989) States not obligated to allow public facilities or public employees to be used for non-therapeutic abortions Court did not reach the merits of claim regarding prohibition of abortion counseling by public employees at public facilities; issue was decided later in Rust v. Sullivan, supra

Page | 18

Informed Consent & Waiting Periods Informed consent requirements Regulations allowed if they inform the woman s decision and encourage childbirth. Casey, overruling Thornburgh and Akron I Can require truthful, non-misleading information about Nature of the procedure Risks of abortion and childbirth Probable gestational age of the fetus State may require that information be given by a doctor. Casey, overruling Akron I

Waiting period Casey overruled the ruling in Akron I that waiting periods are per se unconstitutional A reasonable waiting period (typically 24 hours) will probably not constitute an undue burden. Casey State is permitted to favor childbirth Casey did not rule out a factual showing that a particular waiting period might constitute an undue burden Spousal and Parental Notification or Consent Spousal notification or consent Spousal notification or consent requirements pose an undue burden on the abortion right. Casey, Ashcroft The spousal notification requirement is thus likely to prevent a significant number of women from obtaining an abortion. [F]or many women, it will impose a substantial obstacle. Casey Parental notification or consent The state may require that one or both parents or a guardian be notified of, or give consent to, an abortion for a minor, but only if there is a judicial bypass procedure that allows a court to approve the abortion upon a finding that either: The minor is capable of making the decision; or Having abortion would be in minor s best interests

Abortion Analytical Model


The right is fundamental 1. Does the regulation of it place an undue burden? a. Yes invalid b. No rational basis test 2. Viability is the key point a. Reasonable pre-viability regulations to protect maternal health do not unduly burden (Physician required, etc.)

Page | 19

b. The state may also regulate pre-viability to further policy of encouraging childbirth in protecting its interest in prenatal or potential life. c. Post-viability, the state can ban abortion completely under its compelling interest in protecting prenatal or potential life. d. Prior to Gonzalez, gov t can ban abortion post-viability w/ exception to maternal health. Now, undecided.

Akron Marks earliest indication of O Connor s of the undue burden standard. Webster Plurality decision, but the case highlights things to come. Specifically within Missouri, they are responsible to aggressively challenging Roe. The Legislature s judgment is that life begins at conception. Under Roe, the Court states that a state can t simply decide on its own when life begins. Rehnquist says that the preamble stating it is inconsequential and doesn t regulate anything. Further, he says that it s up to the state to make that determination as to what it is. Viability testing though is what comes up, requiring it to determine viability. However, the testing involves intrusive procedures. The problem is, that the testing makes abortion more expensive and more difficult and less available. The out for the Court is that the physician is the one who determines whether or not to do the tests. Dissent states requiring tests even when the fetus is not viable but puts the physician in a position of having to do the tests to avoid risk of license revocation.

Page | 20

PROCEDURAL DUE PROCESS CONSTITUTIONAL LAW I


Procedural and substantive due process analysis is distinct don t do one for another. Procedural Analytical Model- 3 step, junction and substantive-procedure Distinct from substantive due process, it assumes the government may lawfully deprive a person of life, liberty, or property only if it follows reasonable procedures to minimize the risk of an erroneous or unfair deprivation. Three Questions 1. Is there a liberty or property interest? 2. Has there been a deprivation? 3. What process is due? What has to be done procedurally at a constitutional requirement to satisfy meet the process? What is a property interest? - Typically created by state or federal law, not constitutionally made - How the property interest is created comes into play in determining the due process required for it o Welfare and other public benefits o Public employment o License to engage in trade or profession What is a liberty interest? - Far more expansive; includes fundamental rights and other liberty interests not subject to elevated scrutiny Tests to determine protected property interest - Current Test: Has the government done something to create an entitlement to a property interest Roth v. Board of Regents Alternative Test: Importance of liberty of property interest to recipient Goldberg v. Kelly; still good law, but not used

Procedural due process only if there has been a deprivation of a life, liberty, or property interest. - Simple negligence is not enough, it is beneath the level required to exhibit a constitutional violation - An insubstantial impairment of an interest will not trigger procedural due process - Do not confuse substantive due process test (rational basis, strict scrutiny) with procedural due process

Page | 21

Procedural Due Process Basis Analytical Model 1. Is there a life, liberty, or property interest? a. No Then no procedural due process limitations b. Yes Go to Question #2 i. Property interests (entitlements) typically created by state or federal law, nothing in the Constitution provides this. Roth. Created by statute and it won t remove unless for proper cause. They can also create by express or implied policy. Custom or practice can also create a property interest Vitek. On this case, they agreed on the grounds that a policy existed of not transferring prisoners without a review process. ii. Defeating an entitlement can be done through employment at will provisions 2. Has there been a deprivation of that interest? a. No The no procedural due process required b. Yes Go to Question #3 3. What type of process is required? a. Pre- or post-deprivation process? i. General rule - Some form of pre-deprivation process needed ii. Post-deprivation Appeal will take on a more for formal process, with a hearing and a judge likely. b. Notice Formality of process i. Full evidentiary hearing versus something less ii. As a general rule, pre-deprivation requirement can be limited to notification, with no pre-hearing 1. Exception is Roth, the Court has never overturned it. iii. Post-deprivation procedure could involve an appeal process, typically an informal hearing. This process will be determined based on the pre-deprivation process and vice versa. c. Hearing Three-part Mathews v Eldridge balancing test i. Nature of the individual Interest How significant is it, what impact would the deprivation have? ii. Risk of deprivation Are the procedures in place enough to prevent an erroneous deprivation is wrong? What will result in the future if a procedure is imposed? iii. Nature of state interest What kind of financial or administrative burden will fall on the state if additional procedural requirements will be needed? Standard defenses would be can t afford, not enough resources to accomplish iv. In Mathews, the court said it must be deferential to good faith decisions of the government officer are adequate to establish a good enough due process.

Page | 22

Importance of the interest 1. Goldberg v. Kelly i. Held that welfare recipients must be given evidentiary hearing before benefits terminated. ii. Termination adjudicates important rights iii. How importance is this benefit to its recipient? And if taken away, what impact will it have? iv. This carried over to other cases 2. Bell v. Burson i. State may decline to create property interest ii. But once created, property interest may be essential to livelihood 3. Problem with gauging interest by importance? Entitlement Board of Regents v. Roth 1. Non-renewal of contract did not impair liberty interest. a. There is none because the Board did not damage his reputation, they just did not renew his contract. b. No damage to reputation, standing or associations c. Nothing prevent professor from seeking other employment 2. No impairment of property interest a. To have property interest, must have more than unilateral expectation b. State did nothing to create an expectation of continued employment c. The Court held that a mere unilateral expectation that a benefit will continue is insufficient. Also, the state was silent. How are entitlements created? 1. Stating benefit only termed for cause Zimmerman 2. Express or implied policy Perry de facto tenure policy, the college had nothing written, but was understood that if you were in place for several years, tenure implied. The court said he was entitled to prove if that de facto policy existed. 3. Custom or practice Vitek prisoner transfers to mental institutions, cannot involuntarily transfer without a hearing because a. State s statutory scheme said so b. But mostly because of a custom and practice of no transfer without a hearing. Defeating liberty or property interests 1. Can a liberty or property interest be defeated by denying procedural rights? a. No Arnett v. Kennedy the nature of the property interest are defined by the procedure that is available to protect. In this case, Rehnquist s plurality opinion supported this. It held that the scope of a substantive right can be limited by the limitations on the procedures which are to be employed in determining that right In other words, must take the bitter with the sweet. b. Yes Cleveland Board of Education v. Loudermill defeats Rehnquist s opinion from Arnett i. The bitter with the sweet approach misconceives the due process guarantee

Page | 23

Property cannot be defined by the procedures provided for its deprivation any more than can life or liberty. iii. Legislature may elect not to confer a property interest in employment but once conferred, due process applies c. Castle Rock restraining order case, mother files complaint for no help from police. Scalia states she had no property interest because of how it was worded. Souter better defines by saying procedures cannot define into a property interest. On dissent, Stevens points out that doing nothing was not a discretion available to the police to do nothing. In actuality, this is a point of the Court not wanting to supervise local law enforcement, which would result if the decided the other way. d. Bishop saw ordinance to be at will employment. Firing for any reason or for no reason at all. No expectation created by the ordinance to continue employment. Bishop is saying there is no procedure based on the at will Is there a liberty interest in reputation? Is one s rep standing alone a cognizable liberty interest? 1. Yes Goss v. Lopez student had liberty interest in reputation, Wisconsin v. Constantineau liquor listing law invalidated. 2. No Paul v. Davis shoplifting charge and police lists him as an active shoplifter, but charges dropped. Rehnquist distinguished this from Constantineau in that case, it wasn t just her reputation, it was her freedom to buy liquor. However, factually, it s the exact same situation. Ultimately, the Court is addressing the risk of allowing the Supreme Court to take over tort law type cases instead of issues of constitutional matters. Since Paul, Court has dismissed many of these claims. Deprivation of entitlements Negligence is not enough County of Sac v Lewis guy killed in police car chase. Negligence not enough because this is more of a tort claim instead of a Constitutional issue. [T]he Constitution does not guarantee due care on the part of state officials; liability for negligently inflicted harm is categorically beneath the threshold of constitutional due process. (emphasis added) 1. The standard is whether the conduct shocks the conscience conduct unjustifiable by any governmental interest. 2. The decision does leave slim area for consideration. In non-emergent situation ex. Prisoner needing medication and gets denied. In this situation, possible for it to occur Deprivation must be more than insubstantial Goss v. Lopez (1975) (10-day suspension from public school requires pre-deprivation hearing) 10-day suspension is not de minimis; suspension is a serious event for student Length of deprivation is a factor in determining what type of hearing is required, but is not decisive on the issue of whether any hearing is required

ii.

Page | 24

What kind of process is due for deprivation? 1. Notice Under Mullane v. Central Hanover Bank & Trust, notice must 1. Be reasonably calculated to apprise interested parties of pending action 2. Reasonably convey pertinent 3. Afford a reasonable time to appear

May be tailored to the practicalities and peculiarities of the case Id. Prevailing rule is that notice need not inform party of procedures that may or must be followed to challenge deprivation. City of West Covina v. Perkins Notice must advise customer of opportunity to object to billing and procedure for protesting service cutoff. Memphis Light, Gas & Water v. Craft

There are circumstances where notice can be dispensed with 1. Emergency situation likely to let the state act without notice 2. Impossibility or impracticable Hudson v Palmer In this situation, the court looks at post deprivation remedies 2. Hearing Mathews v Eldridge 3 part balancing test 1. What is the private interest at stake? 2. What is the risk of an erroneous deprivation given existing procedures, and how much will additional procedural safeguards minimize the risk of error? 3. What is the government s interest, and what fiscal and administrative burdens would be entailed by additional procedures?

Mathews is deferential Despite balancing of interests, Mathews requires that substantial weight be given to good faith judgments of legislators or administrators that the procedures they have provided assure fair consideration of the claims of individuals. Mathews is used to answer two questions First, when must some hearing be afforded? Second, what kind of hearing must be afforded?

3. Timing of hearing Exceptions to general rule Emergency: pre-deprivation hearing may be avoided when public interest requires immediate action. Ewing v. Mytinger & Casselberry, Inc. (1950) (seizure of mislabeled drug) Others: using Mathews, courts weigh risk of erroneous deprivation against burdens of additional procedures See, e.g., Ingraham v. Wright (1977) (pre-deprivation hearing not required before corporal punishment inflicted; tort exposure minimized risk of erroneous deprivation) 4. What kind of pre-deprivation hearing?

Page | 25

Generally, something less than an evidentiary hearing is sufficient prior to adverse administrative action. Mathews As a general rule, no pre-deprivation evidentiary hearing is required if Adequate notice is given Opportunity to respond pre-deprivation, either in person or in writing Courts consider post-deprivation remedies

Page | 26

EQUAL PROTECTION
A fundamental premise of the law is that similarly situated persons should be treated the same by the law 14th Amendment Equal Protection Clause prohibits states from unlawfully discriminating against those who are similarly situated Equal protection has been reverse incorporated against the federal government via the 5th Amendment Due Process Clause Three tiers of equal protection scrutiny 1. Strict scrutiny is applied to laws that discriminate based on race, national origin, alien status, and to any classification that substantially impairs exercise of a fundamental right 2. Intermediate scrutiny is applied to a law that discriminates based on gender or illegitimacy 3. Rational basis scrutiny is applied to a law that classifies on any other basis Equal protection analysis is essentially grafted over due process analysis Always start analysis by asking whether law classifies persons, explicitly or implicitly If answer is yes, then do equal protection analysis If answer is no, then go to due process analysis Equal protection analysis will send you to due process analysis if classification is not suspect but infringes the exercise of a fundamental right

Equal Protection: Basic Analytical Model

Always start analysis by asking whether law classifies persons, explicitly or implicitly. Carolene Products, Footnote 4 - When a law treats disfavorably national, racial, or religious, it is subject to less deferential review. Discrete and immutable characteristics something unchangeable - Is it appropriate to divide benefits based on these characteristics? - Is sufficient protection given to groups that need protection?

Five questions, with subparts 1. Does the law classify persons? If yes, go to Question 2 If no, go to due process analysis

2. Is the purpose of the law to discriminate based upon the class and does it have a discriminatory impact upon the class? If yes to both, go to Question 3 If no to one or both, go to due process model

Page | 27

3. Is the classification based on race, national origin, or (state) alien status? If yes, then: 3a: Does the government have a compelling interest to justify the classification? If no, then classification violates equal protection If yes, then go to Question 3b 3b: Is the classification necessary or narrowly tailored to meet the government s compelling interest? If no, then classification violates equal protection If yes, then no equal protection violation If no, then go to Question 4 4. Is the classification based on gender or illegitimacy? If yes, then: 4a: Does the government have an important interest to justify the classification? If no, then classification violates equal protection If yes, then go to Question 4b 4b: Is the classification substantially related to the government s important interest? If no, then classification violates equal protection If yes, then no equal protection violation If no, then go to Question 5 5. Does a non-suspect classification impair the exercise of a fundamental right? Use modified due process model to answer 5a: Is it a fundamental right? If no, classification is judged using rational basis test If yes, go to Question 5b 5b: Does the classification substantially impair the exercise of the right? If no, classification is judged using rational basis test If yes, go to Question 5c 5c: Does the government have a compelling interest to justify the classification? If no, then equal protection violation If yes, then go to Question 5d 5d: Is the classification necessary or narrowly tailored to meet that compelling interest? If no, then equal protection violation If yes, then no equal protection violation Make sure to use the rational basis review (below) if the measure makes it all the way through the Equal Protection Model.

Reverse Incorporation Fundamentally, if the state can t discriminate, the federal government can t either. Equal protection applies to federal government through 5th Amendment Due Process Clause. See Bolling v. Sharpe (1954) (barring racial discrimination in D.C. public schools) Should federal government be free to discriminate when states cannot?

Page | 28

Was Bolling true to original intent of Equal Protection clause? Bolling asks whether or not the law is being faithful to the original intent of the 14th. The same Congress that produced the 14th also created segregated schools in DC

How Laws Discriminate 1. Facial discrimination 2. Facially neutral but discriminatory impact No person who is less than 5 6 and 170 lbs may be a police officer 3. Facially neutral but discriminatory application No person may vote who cannot pass a literacy test Yick Wo v. Hopkins (1886) Rational Basis Review Railway Express Agency v. New York (1949) The law goes after a narrow distraction but leaves all other distractions alone and is under inclusive. The Court doesn t see a suspect class nor a fundamental right violation, and the under inclusive nature makes it allowable. Classification has relation to the purpose for which it is made and does not discriminate in a way prohibited by equal protection Court plays the rational basis what if game What of the fact that the city only banned some traffic distractions but allowed others to continue? New Orleans v. Dukes (1976) What is the government interest? Classic articulation of test: Unless the classification involves fundamental rights or is drawn upon inherently suspect distinctions Court presumes the constitutionality of discrimination and only requires classification to be rationally related to legitimate state interest Here again, under-inclusiveness not a problem Government can implement programs step-by-step, eliminating part of problem now and deferring rest until later Here, OK to discriminate between pushcart vendors based on length of time in city Who gets regulated when government draws lines? Would it have mattered here if all (or most) of vendors were black? Or cajun? Or poor?

New York City Transit Auth. v. Beazer (1979) If problem in Railway Express and Dukes is that legislature did not go far enough, how would you describe problem here? Cf. United States Dept. of Agriculture v. Moreno (1973) (Food Stamp Act so under-inclusive and over-inclusive as to be wholly without any rational basis. ) United States R.R. Retirement Bd. v. Fritz (1980) Blanket exclusion of recovering methadone addicts in employment positions. NYC argues safety. Contrary to Railway and New Orleans, this is OVERLY inclusive. Even so, non-suspect classification, employment is not a fundamental right, rational basis review only and it passes.

Page | 29

The legislature cannot be challenged on whether it understands the law it is passing, and the courts cannot question Congress on those grounds. Must legislature articulate its purpose? [T]his Court has never insisted that a legislative body articulate its reasons for enacting a statute. Court played the what-if game Notice deference to the political process Was Congress misled, or did it misunderstand? Court assumes legislature intended what it enacted Fact you lose a political battle doesn t mean courts will protect you Overly or under inclusive law on its own won t automatically bar a law.

FCC v. Beach Communications (1993) Statutes going to matters of social and economic policy entitled to a strong presumption of validity Party attacking such a statute must negate every conceivable basis supporting it The case shows the limit of the Court deference under rational basis review Are there any impermissible purposes? Not many [A] bare desire to harm a politically unpopular group cannot constitute a legitimate governmental interest. Moreno See also Cleburne v. Cleburne Living Center (1985) ( irrational prejudice against the mentally retarded ); Romer v. Evans (1996) (quoting Moreno and finding classification born of animosity towards class)

EQUAL PROTECTION (CONTINUED)


Equal Protection Suspect Classifications - Race Certain classifications are deemed suspect and are treated to strict judicial scrutiny because their use is rarely grounded in a legitimate governmental purpose Traced doctrinally to Carolene Products FN 4 Suspect classifications subjected to strict scrutiny: race, national origin, and alien status classifications by states Reason for suspect class is there is no legitimate reason to discriminate. Prima facie equal protection violation Must show: The law classifies persons Classification has a disproportionate or disparate impact or effect upon the class Purpose or intent of the law was to cause that disproportionate or disparate impact upon the class To invoke heightened scrutiny, must show that a Suspect Classification, Quasi Suspect Classification, or Fundamental Right is involved

Page | 30

Critical issues Proving discriminatory impact Proving discriminatory purpose or intent Discriminatory impact may be evidence of discriminatory purpose, but is not alone sufficient Other proof (or disproof) of discriminatory purpose Legislative history Facts and circumstances surrounding passage of law Application of law Other circumstantial evidence The easy case: facial discrimination Purpose to discriminate appears facially Assuming law meets intended purpose, prima facie case is made The hard case: facially neutral law Impact or application of law may be easy to prove Difficulty is showing impact or application resulted from discriminatory purpose

Plessy v Ferguson - The equal but separate case. - Laws requiring separate schools, separate accommodations in public conveyances, and banning interracial marriages all upheld - Fourteenth Amendment could not have been intended to abolish distinctions based upon color, or to enforce social, as distinguished from political, equality . (emphasis added) - Facially discriminatory - De jure (mandated by law) discrimination - 19th century civil rights meant rights pertaining to owning land, freely trade, buy and sell but did not include social rights such as attending a theatre, walking down the street. - The Court states the law is powerless to give those social rights - Harlan s dissent was strong against the decision Korematsu v United States [R]estrictions which curtail the civil rights of a single racial group are immediately suspect. . [C]ourts must subject them to the most rigid scrutiny. Did the court apply strict scrutiny? It looks at national security as a compelling interest. It was not narrowly tailored; it affects persons of Japanese descent, citizen or non-citizens inclusive. On this reasoning, it is under inclusive, but overly inclusive to all Japanese. Had the Court used strict scrutiny the law, the law would not have held. Jackson on dissent points out how the law is discriminatory on national origin. Responsibility argument - Act, intent, harm all required before accountability - The action they focus on is being in the wrong place, not national origin, making the law acceptable. Brown v Board of Education Argued twice. Chief Justice Vinson was on the first, but dies while the case is argued. Warren then becomes Chief and case re-argued.

Page | 31

Why the discussion of 14th Amendment history and history of public education? [W]e cannot turn the clock back to 1868 when the Amendment was adopted, or even to 1896 when Plessy was written. Discriminatory impact of segregation In public education, separate but equal has no place and denies equal protection

Facial Discrimination Loving v. Virginia (1967) Rejected claim that equal application of law based on racial classifications prevents application of equal protection Rational basis isn t the test; racial classifications must be subject to most rigid scrutiny and must be necessary to the accomplishment of some permissible state objective There is patently no legitimate overriding purpose independent of invidious racial discrimination which justifies this classification. Restricting freedom to marry solely because of racial classifications violated equal protection Also violated due process

Disparate Application Yick Wo v. Hopkins (1886) Administration so exclusively against a particular class of persons required finding that there was discriminatory intent Though the law itself be fair on its face and impartial in appearance yet, if it is applied and administered so as practically to make unjust and illegal discriminations the denial of equal justice is constitutionally prohibited

Washington v. Davis (1976) Proof of discriminatory impact alone, without proof of a discriminatory purpose, is not enough to sustain an equal protection claim Cases do not hold that a law or other official act, without regard to whether it reflects a racially discriminatory purpose, is unconstitutional solely because it has a racially disproportionate impact. Discriminatory purpose need not be express or facially appear Disproportionate impact can be relevant [A]n invidious discriminatory purpose may often be inferred from the totality of the relevant facts, including [disparate impact]. Disparate impact alone might invalidate a whole range of tax, welfare, public service, regulatory, and licensing statutes Note burden shifting Plaintiff makes out prima facie case; burden of proof shifts to government to show a racially neutral explanation for the result If no prima facie case, rational basis test is applied

Page | 32

Discriminatory Purpose Personnel Administrator v. Feeney (1979) Gender discrimination case Classification based on veteran status, and because it is a non-suspect classification. However, it is seen as a gender classification because of majority of veterans being men. There is a discriminatory impact; however, it does not have discriminatory purpose. The creators of the law likely knew there would be a disadvantage, but that s not enough to establish purpose. It would have to be that they wanted a discriminatory purpose and in this case there is not enough evidence to establish that. The Court sees that the gender discrimination was foreseeable, it can create an inference of discrimination, but it s only the beginning point of making that determination. Discriminatory purpose, however, implies more than intent as volition or intent as awareness of consequences. It implies that a decision maker selected or reaffirmed a particular course of action at least in part because of, not merely in spite of, its adverse effects upon an identifiable group. The inevitability or foreseeability of consequences of a neutral rule may have a bearing on existence of discriminatory intent Where adverse consequences are inevitable, a strong inference that the adverse effects were desired can reasonably be drawn. But such an inference is a working tool and not a synonym for proof.

Arlington Heights v. Metropolitan Housing Dev. Corp. (1977) Discriminatory purpose need not be the sole purpose behind the law When there is proof that a discriminatory purpose has been a motivating factor in the decision, this judicial deference is no longer justified. (emphasis supplied) When determining whether discriminatory purpose was a motivating factor, consider available direct and circumstantial evidence of intent Impact of law may provide an important starting point. A clear pattern, unexplainable on grounds other than race, may emerge from effects of law even when law is facially neutral Historical background of enactment may reveal actions taken for discriminatory purposes Specific sequence of events leading up to enactment Legislative or administrative history may be highly relevant

EQUAL PROTECTION ANALYTICAL MODEL: HYPO


Facts: City of Blurfopolis, doesn t want smoking in bars or restaurants. Studies linking second hand smoke to illness is used as the basis for the ordnance which states there will be no smoking in bars or restaurants within in city limits 1. Does the ordinance classify explicitly or implicitly? - It applies equally to all persons (no one can smoke) No, so go to due process

Page | 33

Due process No long standing tradition of smoking public places. Is smoking implicit in ordered liberty? No so rational basis review. 1. Legitimate interest? Public Health 2. Is it rationally related? Yes Under due process, the ordinance is fine. Under Equal Protection Analysis - By banning the act of smoking, it targets smokers - Does it have a discriminatory impact AND purpose? Impact yes, o Purpose, does one or more legitimate purposes mean that it cannot be discriminatory? No o The standard for determining this is whether it was a motivating factor. Can you argue that a motivating factor was part of the ordinance? We now has a prima facie case on equal protection 1. 2. Classification on gender? No, so no quasi-suspect nor intermediate 3. Non suspect class, so rational basis review in place. The ordinance will pass.

Run Analysis on ordinance - Successful completion of Test 21 is required to become a cop. No classification, there is no equal protections issue, so go to due process o Under due process, employment is not a fundamental right because it is non-textual, nor is there a history or tradition. o It is not implicit in liberty. Under Lochner, it would have been an issue o Under rational basis, the ordinance needs a legitimate interest. Being able to communicate and passing a test to prove it is legitimate. - Does it classify? Yes people that pass the test and those who won t. - In this case, it implicitly classifies on the basis of race. As such, discriminatory impact and purpose needs to be determined. o Impact is established (higher rate of failure among African-American) o When analyzing purpose, begin with the impact. Without the impact, there can t be a purpose. The Yick Wo case shows both impact and purpose. o For establishing discriminatory purpose, there has to be a motivating factor. The Court has never identified what an adequate amount of a motivating factor exactly is.

Page | 34

Equal Protection: Affirmative Action and Other Remedies (week 6 slides) The government may, and in some circumstances it must, act to remedy the effects of past racial discrimination. But the extent to which government may act other than for remedial purposes is unclear Two categories of cases School segregation cases All others involving affirmative

Federal and state governments may act to remedy effects of past discrimination Court has been particularly demanding, especially with states, that showing of past discrimination be made to justify remedy One exception not limited to remedial purposes is public college and university admissions

School desegregation Not heavily tested in the bar because there is no more interest in it. Back in the 60 s it was, today it s not. The Court sees that segregation still exists, but it hasn t been done by anything the states have done at this point.

Doctrine is limited to public elementary and secondary schools that have in the past been segregated

Brown v Board of Education (1955) (Brown II) A year after Brown I which dealt with liability, Brown II is the actual remedy to fix desegregation. The Court sees that the district courts are the proper powers to enter remedial desegregation. The Court spelled out what to look at. Considered the appropriate remedy after Brown I Did not adopt uniform, nationwide standard Authorized local courts to consider Physical facilities Transportation systems Staffing issues Attendance zones

There isn t clarity as to whether the requirement was to formally state the end of desegregation or take affirmative steps to encourage the integration

Green v County School Board answers this, and points out the need for an affirmative duty to integrate. Provided the answer to the desegregation versus integration question Court ruled that [t]he transition to a unitary, non-racial system of public education was and is the ultimate end

Page | 35

School boards were charged with the affirmative duty to take whatever steps might be necessary to convert to a unitary system

Swann v Charlotte-Mecklenburg Bd. Of Ed. In order to invoke the remedial order, there needs to be a showing of a constitutional violation. Equitable remedial powers are broad and include: Limited use of ratios matching attendance with populations in district Altering attendance zones Busing

Keyes v School Dist Answers how to show segregative purpose. The Court created the Keyes Presumption that assists in establishing purpose. If you can show segregative purpose in one section, the Court will impute segregation to the rest of the district. Example. Out of an entire district, only one area receives all the benefit [A] finding of intentionally segregative school board actions in a meaningful portion of a school system [creates] a prima facie case of unlawful segregative design on the part of school authorities, and shifts to those authorities the burden of proving that other segregated schools within the system are not also the result of intentionally segregative actions. If presumption applies, burden on district Not enough to simply show a logical, race-neutral explanation Must prove that segregative intent was not among the factors that motivated their actions. If actions were to any degree motivated by segregative intent and segregation continues to exist, remoteness in time does not defeat inference of intent to discriminate

Miliken v Bradley The Court holds that an inter-district Keyes Presumption violation can t be established unless the segregation in one district directly caused segregation in the other district No inter-district Keyes presumption Must be a showing that something done by Detroit schools effected segregation in adjacent schools Or must show adjacent schools not unitary schools Absent a showing of a violation by a school district it must first be shown that there has been a constitutional violation within one district [producing] a significant segregative effect in [another].

Columbus Board of Educ. v. Penick (1979) Proof that segregated schools have been purposefully and effectively maintained in part of a school district is prima facie proof of a dual school system [A]ctions having foreseeable and anticipated disparate impact are relevant evidence to prove the ultimate fact, forbidden purpose. Use impact as starting point to show purpose

Page | 36

Limitation of Remedy Pasadena City Bd. v Spangler When the objectives of the desegregation have happened, what happens next? The requirements are dropped, and segregation occurred again because of moving and demographics. The order cannot be reinstated, and a new filing has to be done. Issue: where migratory patterns not attributable to segregative actions can attendance zones be annually readjusted? No. Once a racially-neutral attendance pattern is implemented, school board has provided appropriate remedy for previous racially discriminatory attendance pattern

Oklahoma v Dowell Once good faith adherence to the order is done, the order should be lifted. Desegregation order should be dissolved once district has complied in good faith and effects of past discrimination eliminated to extent practicable Thereafter, further efforts to desegregate evaluated under regular equal protection principles

Freeman v Pitts Partial compliance, the order can be lifted in certain areas, and maintain supervision in areas where compliance hasn t been achieved. The Court sees that over time it is less likely that racially segregated schools do not exist because of what the states are doing. What occurred was the creation of suburbia and white flight. The Court in this case sees these issues as local matters, as opposed to the earlier Court. Issue: can a desegregation order be lifted upon compliance with certain parts of the order, but not with others? Yes. A court may withdraw supervision as to those categories where there has been full compliance, but may retain jurisdiction to supervise categories where compliance not yet achieved As the de jure violation becomes more remote in time and these demographic changes intervene, it becomes less likely that a current racial imbalance is a vestige of the prior de jure system. [R]eturning schools to the control of local authorities at the earliest practicable date is essential to restoring true accountability

Affirmative Action Two key categories 1. Bakke, Grutter, Gratz, and Parents Involved in school admission cases y Government not limited to remedial purposes y Diversity is a compelling state interest 2. Wygant, Croson and Adarand in all others (employment, contracting) y Government is limited to remedial purposes y The only accepted compelling interest is remedying the effects of past racial discrimination y Croson points out that strict scrutiny is required for all governments in racial classification measures. y Also, proving past racial discrimination is needed on an institutional basis.

Page | 37

Wygant Strict scrutiny applies No compelling interest in remedying societal discrimination Suggested that remedying past institutional discrimination would be the only allowable justification for race-based remedies Same outcome post-Grutter?

Croson Plan with a requirement in contracting to minorities. The Court points out the federal government can do this under 5 14th Amendment. The reason for strict scrutiny is that while it benefits the minority, it is harming the majority. Also, there is no clear dividing line to distinguish between a benign action and one that isn t. As a result, the Court flat out throws down that ANY race based classification must be subjected to strict scrutiny. Also, the Court sees that remedying a societal discrimination is not a compelling state interest, and the reason for this that if it is allowed there is no stopping point. And within an industry specific societal discrimination alone, there is also no stopping point that would enable it to be a legitimate state interest. On the dissent, Marshall thinks intermediate scrutiny is sufficient because the plan is for a good purpose and a benign classification O Connor s note points to the dissent opinion being deferential to the point of having to Appellant [relies] heavily on Fullilove for the proposition that a city council, like Congress, need not make specific findings of discrimination to engage in race-conscious relief. What appellant ignores is that Congress, unlike any State or political subdivision, has a specific constitutional mandate to enforce the dictates of the Fourteenth Amendment. States and localities do have authority to eliminate effects of private discrimination within their borders Findings of discrimination within an entire industry not enough Nationwide discrimination, either alone or in an entire industry, not enough No evidence of identified discrimination in Richmond construction industry Court reaffirmed that strict scrutiny applies to state racial classifications Calling classifications benign or remedial does not avoid strict scrutiny Classifications based on race carry a danger of stigmatic harm. Unless they are strictly reserved for remedial settings, they may in fact promote notions of racial inferiority and lead to a politics of racial hostility. (Emphasis supplied) Dissenting, Justice Marshall argued for application of a more deferential standard Substantial relationship to important state interest standard Used for intermediate scrutiny Importance of not subjecting to strict scrutiny cities that have come under minority leadership and that seek to rectify past discrimination Note Justice O Connor s response [O]nce the remedial conclusion is reached, the dissent s standard is singularly deferential, and bears little resemblance to the close examination of legislative purpose we have engaged in when reviewing [racial] classifications . Consider this response when reading the Grutter decision

Page | 38

Adarand Looks at a federally based plan that has benign discrimination, and whether intermediate scrutiny is needed. It s not used in this case, because it is too dangerous for a race based classification to be used by any government, federal or state. This case extends Croson s ruling over to the federal government, even with 5 14th Amendment power. [W]e hold today that all racial classifications, imposed by whatever federal, state, or local governmental actor, must be analyzed by a reviewing court under strict scrutiny. Again declined to apply different level of scrutiny to benign racial classifications Consider when reading Grutter

Bakke Looks at UC Davis admission program. 16 spots reserved for minorities only. The purpose of having minorities in the class with the assumption that those students will practice in those underrepresented minority areas. The Court points out that a quota program is not allowed under any measure. Plurality decision, Powell only accepts having a diverse student body as a legitimate compelling interest. However, having this interest requires proving that racial diversity on its own is insufficient. Each applicant needs to be evaluated in an individual basis, the individualized consideration of the applicant s potential to contribute to a class.

The Court concluded Davis was not interested in a balance

Rules of law coming from Bakke y Strict scrutiny failed because a quota system is not narrowly tailored y Substantial interest in diversity requires more than just racial diversity Justice Powell cast the 5th vote and his is generally (though not universally) considered the opinion with precedential force Strict scrutiny When a person is classified on basis of race or ethnicity, he is entitled to a determination that the burden he is asked to bear is precisely tailored to serve a compelling governmental interest. Compelling interests Justice Powell rejected most of those offered Purpose to assure percentage representation of minorities (e.g., a quota) is facially invalid State has legitimate and substantial interest in eliminating effects of identified discrimination But purpose to remedy societal discrimination is not sufficient Attaining a diverse student body can be a compelling interest Narrow tailoring Program focused solely on racial diversity would hinder rather than further attainment of genuine diversity. Program must treat each applicant as an individual in the admission process

Page | 39

No facial infirmity exists when race is but one factor and is weighed against other factors

Justice Powell s opinion is generally cited for two propositions First, that the admissions program failed the test of strict scrutiny because its quota system was not narrowly tailored Second, that a state has a substantial interest in student body diversity that may be served by a competitive admissions program that properly considers race and ethnicity

Grutter v. Bollinger Law school applicant to UM declined despite having better admission numbers. UM doesn t say anything about remedying past discrimination, which is not something that will be readily admitted. UM states that their interest is a diverse student body. Looking at Croson and Adarand, it states the only reason for racial classification is past discrimination. However, O Connor states that prior holdings only suggested that not required.

The school s goal of critical mass is never defined, which gives rise to whether the term is merely a different name for a quota. O Connor s opinion mentions that the school s judgment to determine the educational mission is who they defer to, assuming good faith of the institution. In this example, strict scrutiny is likely not used, because there is nothing to show Strict scrutiny remains the standard Justice O Connor clarified prior holdings suggesting that only justification for racial classifications is remedying past discrimination [W]e have never held that the only governmental use of race that can survive strict scrutiny is remedying past discrimination. [W]e hold that the Law School has a compelling interest in attaining a diverse student body. The Law School s educational judgment that such diversity is essential to its educational mission is one to which we defer. Further, good faith on the part of a university will be presumed absent showing to contrary Is this strict scrutiny? Court found that Law School s admissions program was narrowly tailored Not a fixed number or percentage Goal of critical mass does not make it a quota Race is merely used as a plus factor Each applicant is reviewed individually Are there less restrictive means available? Narrow tailoring does not require exhaustion of every conceivable race neutral alternative. Why is diversity a compelling interest here? What makes diversity more important here than in the workplace? Or anyplace else?

Page | 40

Is the state concerned with something more in an educational setting than it might be in a workplace? Is diversity forward-looking or backward-looking?

Parents Involved In Community Schools v. Seattle School Dist. (PIICS case) Looked at measure of school district that adopted student assignment plans that rely upon race to determine which public schools certain children may attend. At one point in KY there was desegregation was in place but not longer and the Seattle district had none. Their admission plans were completely voluntarily, but focused on minority only. However, the plans were poor in construction and fails narrowly tailoring 1. Focus was only on racial diversity 2. Also, in both instances the schools were already diverse and the impact of the plan was minimal. This plan is seen as under inclusive in that it doesn t affect enough students. The Court does not apply Grutter in this case. Desegregation or affirmative action? School districts never de jure segregated, or had achieved unitary status Plurality applies standard equal protection review; not controlled by desegregation cases Permissible state interests Remedying effects of past discrimination Diversity Remedial interest not applicable here, because either no de jure segregation or unitary status achieved Diversity interest also not applicable here [T]he plans are directed only to racial balance, pure and simple . Student bodies are already substantially diverse, without these admission plans Narrow tailoring Minimal effect racial classifications have suggests that other means would be equally effective No showing that school districts considered other means Justice Kennedy s concurrence Not willing to rule out racial classifications in all cases

Equal Protection: Other Suspect and Quasi-Suspect Classification In addition to race, certain other classifications are suspect or quasi-suspect and result in higher scrutiny Suspect (in addition to race) National origin Alien status General rule for state law classifications based on alien status is strict scrutiny Exception: states may exclude aliens from political functions generally including: Voting

Page | 41

Elected or appointed positions involving creation, execution, or review of policy If a political function, rational basis test is applied Federal classifications based on alien status generally reviewed under rational basis standard Federal government has authority to admit or exclude aliens Court has reasoned that federal government thus has greater discretion to classify based on alien status

Graham v. Richardson (1971) Denial of state welfare benefits based on alien status violated equal protection Aliens are a discrete and insular minority contemplated by Carolene Products FN 4 Really? Always? Classifications based on legal alien status are inherently suspect Subject to close judicial scrutiny Exceptions Undocumented aliens not a suspect class Plyler v. Doe (1982); rational basis review Political functions, sometimes also called governmental functions Rationale: generally, sovereign can establish its own form of government and can limit the right to govern to those who are members of the political community Issue: What constitutes a political function? Elements of political functions exception Applied to persons holding elected or important non-elected legislative, executive, or judicial positions Exercise discretion and participate in formulation, execution, or review of public policy Perform functions going to heart of representative government Have significant contact with or authority over persons

Ambach v. Norwick (1979) State law alienage classification applies strict scrutiny reasoning for this is the Court sees a political function in place. However, because of the ambiguity of the political function it is rational basis. The Court points to the influence that teachers have over their students and can impact their learning regarding the government. However, use of the political function test can be manipulated easily. Further, there is nothing to define a fundamental governmental obligation. The political functions standard is simply not defined well. Public school teaching goes to the heart of representative government Public education, like policing, is a fundamental government obligation Wide discretion in the way material is taught Influence over attitudes toward government, political process, and social responsibilities Test is rational basis Federal government has wider discretion to classify on basis of alien status

Page | 42

May exclude aliens and may establish terms upon which they may enter and remain. Lem Moon Sing v. United States (1895)

Mathews v. Diaz (1976) Congress can make rules applicable to aliens that would be unlawful if applied to citizens Rational basis review applied Quasi-suspect Is the classification substantially related to achieving an important state interest? Other classifications Not labeled suspect or quasi-suspect, but not simply rational basis review Sexual orientation Romer; Lawrence Mental retardation City of Cleburne Age and wealth not suspect classifications Intermediate scrutiny; government bears the burden of having an important interest and a substantial relation exists to meet the purpose. Gender Reed v. Reed Looks at a whether a rational relationship exists (rational basis review) Frontiero v. Richardson Cites Reed, stating that is points to close judicial scrutiny. Reed gets changed and looks like intermediate scrutiny. Plurality claimed that based on Reed, gender classifications are inherently suspect and must be subjected to close judicial scrutiny. Said Reed departed from traditional rational basis scrutiny Craig v. Boren Looks at a beer buying measure differentiating between males and females and limits purchases to 18 for females and 21 for males. Oklahoma bases its rule on statistics reflecting a higher percentage of males driving drunk.

The Court sees no substantial relation between the measure and goal. This is an example of failing on means testing. Analysis [P]revious cases establish that classifications by gender must serve important governmental objectives and must be substantially related to achievement of those objectives. Accepted state purpose to enhance traffic safety Statute failed on means testing: if maleness was a proxy for drinking and driving, a correlation of 2% was an unduly tenuous fit.

Page | 43

US v. Virginia VMI all male school case. Virginia states an interest in educational diversity to justify VMI s policy. The Court questions where this opportunity exists for women. Not only must the interest be important, but must show an exceedingly persuasive justification.

Adding the exceedingly persuasive interest is an attempt to raise the gender classification to a level of strict scrutiny. So far, this has been mentioned only on gender classification as quasi-suspect only and intermediate scrutiny is the only requirement. (need to mention this factor in an essay question) Virginia s VWIL is a weak response, and the Court points out that it is close to separate but equal program.

The Court points to the following as justification for gender based classification: 1. Compensation for economic disabilities suffered or 2. Promote equal employment opportunity Court recited intermediate scrutiny standard Burden of justification is demanding and rests entirely on the State. State must show exceedingly persuasive justification Permitted reasons for gender classifications To compensate women for economic disabilities suffered Promote equal employment opportunity State failed to provide exceedingly persuasive justification so equal protection violated Proposed remedy unacceptable Parallel program for women at another state university Proposed program was not the equal of VMI; reminiscent of Sweatt separate but equal Justice Scalia, dissenting Court s decision today will have consequences that extend far beyond the parties to the case. What kind of argument do you know is coming? Single-sex education is unconstitutional Government may violate equal protection by funding single-sex education Recall Justice Souter s concurrence in Glucksberg: when attacking a rule, how do you characterize it?

NOTE: On a gender classification case essay question MUST include exceedingly persuasive justification in the discussion in addition to equal protection discussion. This term is only used in gender cases. Illegitimacy cases haven t used it. Geduldig v. Aiello Excluded pregnancy related conditions from disability benefits. Gender discrimination against women, but the Court looks at pregnant and non-pregnant persons.

Page | 44

The false classification dilemma comes out, pointing out that not all women will be pregnant, and it is classifying not on gender but on disability (pregnancy) and there is nothing requiring strict scrutiny on those grounds. The Court is saying don t confuse a condition unique to a class to one that must be experienced by the entire class. Califano v. Webster Remedial in nature. In the race context, proof of direct involvement in past discrimination required. In gender situations, it is not as stringent. Reduction of the disparity in economic condition between men and women caused by the long history of discrimination against women has been recognized as an important governmental objective. Court will strike such statutes when they are not enacted as compensation for past discrimination Recitation of remedial purpose will not prevent Court from looking to actual purpose

Miss. Univ. for Women v. Hogan Female-only nursing school. The school justifies it on the grounds that it is compensation for discrimination against women. However, there is nothing showing that discrimination in nursing and only perpetuates the stereotype that women are only meant to be nursing. The case highlights the need to remediate something that actually needs it. Compensatory purpose can be invoked only if members of class benefited actually are at a disadvantage related to classification State failed to show that women lacked opportunities in nursing Alleged benign purpose (remedying past discrimination) was not actual purpose

Illegitimacy Mathews v. Lucas Today, not very applicable because of culture However, in dealing with children, decisions look at children not being responsible for the actions of their parents. And because of that, laws against parents of an illegitimate child will be looked at only with rational basis review. Though not clear, Court suggested standard was something between rational basis and strict scrutiny Under the standard of review appropriate here which Court later described as this realm of less than strictest scrutiny The scrutiny was not a toothless one Clark v. Jeter (1988) confirmed that intermediate scrutiny is the standard Who is the victim of discrimination?

Page | 45

Cases suggest discrimination against parents may be subject only to rational basis review Glona v. American Guar. & Liab. Ins. Co. (1968) (striking state law denying mother wrongful death recovery for death of illegitimate child) Califano v. Boles (1979) (upholding federal law denying benefits to mother of illegitimate child) Justification for treating the parents to a different standard of review?

Other Classifications Mental Retardation Cleburne v. Cleburne Living Center zoning ordinance ordinary economic/social legislation, and because of this the presumption is that it will be constitutional and faces rational basis review. The City applied the ordinance specifically against the living center. The Court says that they are not quasi-suspect or suspect based on the following: 1. Those mentally retarded have a reduced ability to cope with and function in the everyday world. (not a job for judges to determine their needs institutional competence argument) Not excluded, They have been treated to attention in the political system Lack of formal realizability recognizing them as quasi-suspect makes other groups difficult to draw the line.

2. 3. 4.

The Court sees that the reasons of the City for the ordinance are not a legitimate state interest and is an irrational prejudice and invalid. But the Court isn t really using rational basis review; the City s reasons are arguably legitimate. Cleburne stands for the mentally retarded simply for the reason of discrimination is irrational. Also, looks at the handicapped as a whole are not a quasi-suspect leading to no strict scrutiny. FN 24 points out the young being barred but it not being immutable. Mental retardation not a quasi-suspect classification Some objectives, like bare desire to harm politically unpopular group, are not legitimate state interests Classification rested on irrational prejudice against mentally retarded Rational basis was the standard but is that really what the Court applied?

Page | 46

Sexual Orientation Romer v. Evans Colorado constitutional amendment barred policy or practice to protect homosexuals, lesbians, or bi-sexual. The state argues that they don t want a group to have special privileges. The Court points out that the measure is to deny a disfavored group the benefit of laws that everyone else enjoys. This is a rhetorical argument Using what the Court calls rational basis review, the Court says that a legitimate interest exists in regard to conserving finances, but still rejects the amendment. The enactment is a status based measure, unlike Bowers barring on action (homosexual activity), and punishment is based on who and what you are. What the amendment is doing is punishing before any responsibility for a wrong has been done. The measure also excludes a group entirely from the political process. What was one key difference between this amendment and statutes at issue in Bowers and Lawrence? [G]overnment [must] remain open on impartial terms to all who seek its assistance. Disadvantage was born of animosity toward the class of persons affected. [A] bare desire to harm a politically unpopular group is not a legitimate interest

Non-Suspect Classifications Plyler v. Doe (1982) Undocumented aliens not a suspect class, and education not a fundamental right No gender or illegitimacy classes But statute imposed a lifetime of hardship on a discrete class of children not accountable for their disabling status. Court applied an intermediate-type scrutiny, requiring showing of substantial state interest Age is not a suspect classification. Massachusetts Bd. of Retirement v. Murgia (1976) Wealth or lack of wealth, standing alone, not a suspect classification. See Harris v. McRae (1980)

Where fact hypo has a group claiming quasi-status, use factors from Cleburne discrete or insular? Politically powerless? Despite several request for suspect classification, the Court has not recognized many new ones.

Page | 47

Equal Protection - Voting Rights There is a fundamental right to vote which is protected by the equal protection guarantees of the Fifth and Fourteenth Amendments, as well as other amendments to the Constitution

A classification which selectively distributes or impairs the right to vote will be strictly scrutinized Statutory laws have essentially limited any major changes in voting rights.

Voting rights and the constitution is typically about equal protection because the constitution does not guarantee the right to vote in state elections. It protects the equal right to vote. Fifth and Fourteenth Amendments guarantee equal right to vote In addition, other Amendments speak to voting rights 15th Amendment (race) 19th Amendment (sex) 24th Amendment (failure to pay tax) 26th Amendment (18 and older)

If a state grants a right to vote, it must extend the right to all qualified voters under the 5th and 14th Amendments Voting rights cases may be categorized into 5 groups 1. Selective denial of voting rights 2. Individual vote dilution ( one person, one vote ) 3. Group vote dilution or gerrymandering 4. Majority-minority districts 5. Vote counting

Groups of voting rights cases 1. Selective Denial Kramer v Union Free School Dist. Any effort to selectively denial voting rights will be subject to strict scrutiny. Residence, Age, Citizenship are the criteria the state can use to determine the right to vote. Anything else will have to go through strict scrutiny. Strict scrutiny: [I]f a [statute] grants the right to vote to some and denies the franchise to others, the Court must determine whether the exclusions are necessary to promote a compelling state interest. Willing to assume for the sake of argument that state might limit electorate to those who are primarily interested in school affairs How can the state classify potential voters? [A]ny classification restricting the franchise on grounds other than residence, age, and citizenship cannot stand unless the State can demonstrate that the classification serves a compelling state interest. Hill v. Stone (1975) (emphasis supplied); see also Kramer Individual vote dilution Legitimate factors (Karcher v. Daggett (1983)): Compactness and contiguity Respecting municipal boundaries

2.

Page | 48

Avoiding contests between incumbents Illegitimate factors (Reynolds): History (standing alone) Geographic size Economic or group interests

Reynolds v. Sims (1964) Leading case on one person, one vote We hold that, as a basic constitutional standard, the Equal Protection Clause requires that the seats in both houses of a bicameral state legislature must be apportioned on a population basis. States must make honest, good faith effort to equalize populations Deviations based on legitimate considerations incident to the effectuation of a rational state policy are allowed Reapportionment every 10 years based on U.S. Decennial Census appears to be a rational approach to readjustment of legislative representation . Less frequent readjustment would assuredly be constitutionally suspect. The one person one vote is shown in the Reynolds v Sims case meaning each vote carries the same weight as the other. Voting districts and the determination of them take a role in this, where each district must be equalized by a good faith effort

Example A 10k B 20K C 30k The county commission is a 3 member board each going to each district. Therefore, A s commissioner will only have the third of the power C s has. To remedy this, the number of commissioners will be increased to equal the ration (1 commissioner per 10k voters) Also, the districts can be rezoned to make an equal amount of voters in each district The Court sees that this must be done on an rough basis, meaning the balancing does not have to be exact. Also, re-districting must be looked at every 10 years For state and local elections, Reynolds simply requires that there be substantial equality in district sizes so that each vote is approximately the same weight as all others De minimis or minor deviations of as much as 10% seem to be tolerated; such deviations do not make out a prima facie case of dilution and do not require justification and larger deviations may be justified. Gaffney v. Cummings (1973) No de minimis exception for federal districts. Karcher Standard for any deviation is strict scrutiny. Id. Prima facie case is made by showing that a smaller deviation was possible. Id.

Page | 49

Upon prima facie showing, burden shifts to state to show deviations were necessary to achieve some legitimate state objective. Id.

3.

Group vote dilution or gerrymandering

Group vote dilution was done to draw district lines in a way that made a group insubstantial. Today, political dilution is done by both parties. Meaning, by having a majority in the house, that party can redistrict to their advantage. The Court allows this to happen. Group vote dilution is possible when any voting minority is present in a political subdivision Often the minority is racial, but it need not be Accomplished in three ways: 1. At-large elections 2. Gerrymandering 3. Multi-member voting districts In all three instances, electoral districts are designed in such a way that minority group will not have a majority in any electoral district Equal protection claims of group vote dilution based on at-large, gerrymandered, or multimember districts require proof of both discriminatory purpose and effect

Davis v Bandemer In political gerrymander case, plaintiffs required to prove intentional discrimination against identifiable political group and discriminatory effect on that group But showing of discriminatory purpose and effect alone not enough There is no equal protection right to proportional group representation Fact that minority group simply fails to get its representatives elected in proportion to minority group s ratio to total population is not enough [W]e have required a substantially greater showing of adverse effects than a mere lack of proportional representation to support a finding of unconstitutional vote dilution. Bandemer

There is an attempt to stop redistricting. A discriminatory purpose and impact exists in doing this. But the court sees despite the establishment of discriminatory impact and purpose, in voting cases there needs to be even more. The Court needs to see be shown a complete shut out of voting for a particular group, which has never been proven.

Veith v. Jubelirer Four members of the Court see that voting is non-justiciable. But Kennedy doesn t see it that way, and states that he can t see a justiciable case in this specific case, but he won t say outright that all voting cases are non-justiciable.

Page | 50

4.

Majority-minority districting Involves redrawing boundaries to maintain a voting minority group as the majority.

The Voting Rights Act 5 pre-clearance requirement amendment of 1965 mandates redrawing based on race. Facially, it looks like an equal protection issue and a constitutional problem and as such it is subject to strict scrutiny. However, it must show that race was the predominant factor motivating the drawing of a district. Miller v Johnson

In Shaw v Reno, the shape of the district alone can establish a prima facie case for race-based districting. Meaning, the district is so misshapen the purpose of it is obvious and subject to strict scrutiny. Shaw suggests that irregularly-shaped district alone may establish prima facie case of unconstitutional race-based districting If prima facie race-based districting case established, standard is strict scrutiny State must show compelling interest and narrow tailoring

5.

Vote counting Bush v Gore whatever irrelevant case with no precedent set and no opinion. Case is clearly significant for political reasons Meaningless from a doctrinal standpoint Any impact on legitimacy of Court or legacy of Rehnquist Court?

Page | 51

FREEDOM OF SPEECH
Since the turn of the 20th Century, the Court hasn t been methodical in putting measures in place regarding free speech decisions. The 1st Amendment favors free speech because the Framers knew the natural tendency of government is to suppress speech.
y y

The normal assumption is that the government acts in the public interest. In Reality: The 1st Amendment is contrary to that, and when the government regulates speech its acting against the public interest.

Use these points as a guideline in how the Court makes its decisions regarding free speech. The basic dividing line in free speech doctrine is content regulation. y If the measure is content based, strict scrutiny review will be done. y If it is content neutral, a form of intermediate scrutiny will apply. The government bears the burden of proving content neutral basis and does not need strict scrutiny Start with the premise that government wants to regulate speech can it? Yes, but it must be careful (the government will try to establish a law that will avoid strict scrutiny). Content regulation is the fundamental fault line - Content regulation includes subject matter and viewpoints - The general rule is that content-based regulations of speech are presumed invalid and must satisfy strict scrutiny. Compelling state interest, narrowly tailored, using least restrictive means - Content involves subjects and viewpoints and an argument to which is what will result. Ex. War is good, war is bad. Subject (war) viewpoint (good or bad) How does the government avoid strict scrutiny? Four ways: First, by fitting into an unprotected category, (Fighting words, obscenity, etc.) it can be banned - Inciting illegal activity - Fighting Words - Obscenity - Child pornography - Defamation Otherwise, it has to be content neutral. Second, fitting the speech into a category the government can regulate These categories are narrow, but do enable rational basis review if possible. - Commercial speech: an intermediate form of scrutiny - Speech at schools, prisons, and military installations: rational basis - Traditional broadcast media: government can ban indecent content

Page | 52

Other non-public and limited designated public forums: regulations must be reasonable and viewpoint neutral

Third, regulate conduct, not speech (O Brien case) - Government can ban conduct if the government does not target expression and the incidental burden on speech is no greater than essential - Intermediate form of scrutiny used - The conduct v. content issue comes up here Fourth, regulating the time, place, or manner of speech - Say what you want, just not at this time, right here, or in this way - Intermediate form of scrutiny - Example free speech zones at political conventions If the government can t fit into one of the four, it is deemed content based and strict scrutiny will apply. Note that special rules apply when the government either: - Subsidizes speech (public libraries have content based decisions) - Is the speaker, not a regulator (White House news conferences) and can make its own message - Government cannot also establish a religion or express one (Establishment clause) And the review will be closer to rational basis. The argument against government regulation will always be after the content itself, even if the government uses other rules such as location, time Unprotected Speech A few categories of speech often described as unprotected can, as a general rule, be completely banned which include: Inciting illegal activities and hostile audiences Current test is Brandenburg v. Ohio: speech intended to have effect of producing imminent illegal action and likely to produce such action Under hostile audiences doctrine, speech that creates a clear and present danger of riot, disorder or similar threats to public health and safety can be regulated. Feiner v. New York

Obscenity Current test is Miller v. California three-part inquiry 1. Average person applying contemporary community standards would find work as a whole appeals to prurient interests 2. Work depicts or describes in patently offensive way sexual conduct defined by applicable law 3. Work taken as a whole lacks serious literary, artistic, political, or scientific value Fighting words Fighting words are those which by their very nature inflict injury or tend to promote an immediate breach of the peace. Chaplinsky v. New Hampshire

Page | 53

Child pornography States may ban visual pornographic depictions, Ferber, but must involve actual children. (adults depicting children is not child pornography, digitally created images also not) Ashcroft v. Free Speech Coalition Defamation

Other kinds that facially would fit in a category (racist speech, profanity) are likely to be protected - Government cannot ban offensive, profane or racist speech outright as it can unprotected speech, but can try to regulate in a variety of ways - The presumptive remedy for offensive speech is more speech to argue it out Inciting Illegal Activity Cases reflect historical reference only, free speech never an issue until this point.

Schenck v. United States (1919) The words are used in such circumstances and are of such a nature as to create a clear and present danger that they will bring about [harm]. Debs v. United States (1919) Purpose to incite others to violate the law Reasonable probable effect of inciting violation of law

The Espionage Acts were in response to World War I, and the cases regarded persons speaking out against the government. Out of it comes Holmes clear and present danger doctrine. It was used in a restrictive way. Not until after the Communist scare of the 1950 s did the doctrine begin to relax. What the doctrine was criticized for was punishment for advocacy. In Yates, the Court begins to recognize that advocacy alone can t be banned, it must be more to incite action. Dennis v. United States (1951) Question is whether the gravity of the harm, discounted by its improbability, justifies regulation of the speech to avoid the danger Existence of harm is a question of law Court said it applied clear and present danger test did it? Government need not wait until threat is imminent before it acts Cf. Yates v. United States (1957) The essential distinction is that those to whom the advocacy is addressed must be urged to do something rather than merely to believe in something. (emphasis in original) Brandenburg v. Ohio (1969) The case was prosecuted under Ohio s syndicalism act which worked against violent overthrow. Acts banning violent overthrow easily enforceable because of the democratic process of being able to peacefully change the government.

Page | 54

The Court makes the incitement standard with this case May not forbid or proscribe advocacy unless Advocacy is directed to inciting or producing imminent illegal action; and Advocacy is likely to incite or produce such action This is the modern restatement of the clear and present danger rule Note similarities to Debs predecessor Directed to inciting illegal action (purpose) Likely to produce such action (effect) Key element: imminent illegal action Mere advocacy alone is not enough; must incite to imminent illegal action Even though Brandenburg still stands, holding on bans is more difficult today. Hess v. Indiana points out how there is nothing said in the statement reflecting imminent illegal action (we ll take the street later).

Hostile audiences Feiner v. New York (1951): [W]hen as here the speaker passes the bounds of argument or persuasion and undertakes incitement to riot . Black s dissent point out that free speech is for the speaker, not the crowd. The person making his speech was arrested on the grounds that a clear and present danger and under those grounds police power can be done. Hostile audiences doctrine seems to have been narrowed Gregory v. City of Chicago (1969): conviction overturned because ordinance not limited to threats of imminent violence

Obscenity Miller v. California Obscenity limited to works that depict or describe sexual conduct Sexual conduct must be specifically defined by applicable state law The Miller test looks at the criteria for what is obscene and includes: 1. Whether the average person finds that it appeals to a prurient interest 2. Whether the work depicts patently offensive sexual conduct as defined by the law (no strict 3. Whether the work as a whole has no serious literary, artistic, political or scientific value
y y

The first two are contemporary community standards which means the standard is not determined by the extreme, it s the average Although patent offensiveness is a community standard, there is a national floor that a community can t go below. Jenkins v. Georgia (1974) found the film Carnal Knowledge not patently offensive It did not depict or describe patently offensive hard core sexual conduct. Some nudity, but nudity alone was not enough to make material legally obscene under the Miller standards.

Page | 55

The third is a national standard Under Pope v. Illinois (1987), literary, artistic, scientific, or political value is judged by a national standard Miller is intended to capture hard core pornography even though that is difficult to define. This is a proactive standard, meaning that these issues can be brought up, but they don t have to be.

Stanley v. Georgia Established the possession of pornography as legal, but the government can ban the shipment or mailing of it. Court has repeatedly declined to extend Stanley Paris Adult Theater I v. Slaton (1973) State has legitimate interests in minimizing commercial obscenity See also United States v. Reidel (1971) (Stanley does not protect mailing of obscene materials to adults) United States v. Orito (1973) (Stanley not extended to transporting of obscene materials) There is a distinction between obscenity and porn. Obscenity is a subset of pornography, meaning not all pornography is obscene.

Overbreadth The general rule is that you can t argue that the rule is overbroad. Meaning you can challenge a statute as applied to you specifically, but you can t argue that it applies to third parties, a facial challenge. Which means the statute can t be applied to anyone. But the following exception can apply in 1st Amendment issues. Ex. Arrest for disturbing the peace. Defense is the law is vague and overbroad Party lacks standing to challenge constitutionality of statute as applied to others. Broadrick v. Oklahoma (1973)

Because of importance of free speech, exception made for 1st Amendment issues If a statute is substantially overbroad, a party whose conduct would otherwise be unprotected may raise a challenge

See New York v. Ferber (1982) Discussed First Amendment substantial overbreadth doctrine A speech regulation may be invalidated as substantially overbroad if the regulation, as reasonably construed by the state, reaches a substantial amount of protected speech A single or a few impermissible applications not enough to invalidate Must be substantial number of invalid applications Distinguish facial versus as applied challenges Party s speech is not protected; can still raise substantial overbreadth challenge. Ferber Party s speech is protected; can party also raise substantial overbreadth? Cf. Maryland v. Joseph H. Munson Co. (1984) (suggesting yes) with Brockett v. Spokane Arcades, Inc. (suggesting no)

Page | 56

Substantial Overbreadth doctrine y A statute that is substantially overbroad can be challenged on its face. Even if your speech is unprotected, you have standing to challenge a speech regulation that is substantially overbroad. y Unprotected speech (obscenity) for example, you can argue with the overbreadth doctrine. y If the speech is protected, there is a split whether the overbreadth doctrine can be used.

Vagueness 1st Amendment concern because insufficient notice is the issue and it may have a chilling effect on free speech. As a general proposition, challenging

Also, 5th and 14th AM concerns namely, how can you conform to a law when you don t know what the law requires? Any speech related prosecution will typically raise this as an affirmative defense.

Vague statute regulating speech presents two constitutional issues 1. First Amendment infringement of speech 2. Due process issue relating to adequate notice Vagueness and overbreadth frequently go together, but need not be so. See City of Los Angeles v. Jews for Jesus, Inc. (1987) (ordinance banning all First Amendment activities at LAX overbroad, but not vague) Coates v. City of Cincinnati (1971) Ordinance banned assembly of three or more persons on sidewalks who conducted themselves in a manner annoying to persons passing by . Ordinance was unconstitutionally vague Subjected constitutional right to an unascertainable standard [N]o standard of conduct is specified at all. Also, there is a chilling effect on free speech because it would prevent someone from stating

Child Pornography New York v. Ferber (1982) Prevention of sexual abuse and exploitation of children a state interest of surpassing importance States have a compelling interest in prosecuting those who sexually exploit children Child pornography, like obscenity, is unprotected, but there are constitutional limits Limits (the standard) State law must adequately define conduct, including sexual conduct, to be prohibited Offenses must be limited to works that visually depict sexual conduct by children below a specified age. (emphasis in original) Materials that do not meet Ferber standard retain First Amendment protection No criminal liability absent scienter

Child pornography must involve actual children; government cannot ban digitally created images or depictions of adults portraying minors. Ashcroft v. Free Speech Coalition (2002)

Page | 57

No constitutional right to possess child pornography in the home; compelling reasons exist to prohibit possession. Stanley distinguished. Osborne v. Ohio (1990) With general pornography, its basis is protection from ourselves. In child pornography, its about protection of children The Court held that the government can restrict the sale of child porn even if it doesn t meet the Miller obscenity standards.

Fighting Words Chaplinsky v. New Hampshire (1942) Words which by their very utterance inflict injury or tend to incite an immediate breach of the peace. Chaplinsky is still good law, but Court has never since used it to uphold a conviction Narrowing the doctrine Texas v. Johnson (1989): no one could interpret flag burning as a direct personal insult or an invitation to exchange fisticuffs.

Offensive Speech Indecent and profane speech Leading case is Cohen v. California (1971) - Baliff arrests Cohen for wearing a jacket that was offensive Fuck the Draft for disturbing the peace. State advanced two theories: 1. Language was likely to cause a violent reaction A vague fear of disturbance was insufficient 2. May regulate morality by banning this particular word How do we distinguish this bad word from any other?

The state s argument is that people offended will incite breaches of the peace, which is insufficient. The Court sees that they are not fighting words because they are not recognized as a personal insult to anyone, even though it easily could be. The Court doesn t uphold the ruling probably because it wanted to allow such speech to be given. Further, there is potentially a lack of formal realizability, if you ban one word, what s the next one going to be? From the decision, generally, profane language is protected under the 1st AM. However, a contentneutral time, place, or manner restriction could be put in place. The dissent is trying to move this into intermediate scrutiny by seeing the action as not speech.

Government can regulate indecent and profane speech over traditional broadcast media. FCC v. Pacifica Foundation (1978)

Page | 58

Racist speech R.A.V. v. City of St. Paul (1992) Found that ordinance was content based In theory city could ban all fighting words, but it could not ban particular types of fighting words (such as hate speech) Nor could city ban particular viewpoints R.A.V. should be equally applicable to any kind of unprotected speech e.g., city could not ban homosexual obscenity but permit other kinds Ordinance restricting fighting words. Meaning the city is free to make a content-based decision on what to ban. Scalia states that it is not free to make content based distinctions within the category of fighting words. The distinction applies not only White s concurrence states that you can ban an entire category of speech, but not a smaller category, which is under inclusive. He sees that the rule cannot stand. Content based distinctions within the category of unprotected speech are not allowed. See also Virginia v. Black (2003) Court said a law that simply banned cross burning with intent to intimidate would be permissible and consistent with holding in R.A.V. But prima facie evidence provision rendered statute unconstitutional True that many cross burnings are done with specific intent to intimidate But cross burning may have other symbolic or expressive purpose that cannot be proscribed The true threat standard someone conveys threat to cause harm. The statute presumes that the intent of the action (cross burning) is to threat. If it s a true threat, it s fine. But the presumption of causing harm is not allowable. O Connor discusses how cross burning has had different meaning over time, making the assumption unacceptable.

Symbolic Speech & Time Place or Manner Regulations Sometimes the government wants to regulate speech but it must deny that it is regulating speech. These regulations fall into two categories: 1. Symbolic speech combining speech with other conduct, where government seeks to regulate the conduct but not the expression 2. Time place or manner regulations you can speak, just not here, at this time, in this way Symbolic Speech Regulates speech without saying its regulating it.

Symbolic speech combining speech with other conduct, where government seeks to regulat the conduct but not the expression There are different names to symbolic speech, but the focus is on conduct. May go by different names, including expressive speech or speech-conduct Idea is the same: the speaker intends to use action, or action and words together, to express a message United States v. O Brien is leading case. Government can regulate the conduct if it can articulate a sufficient non-speech interest to justify the incidental burden on expression

Page | 59

United States v. O Brien O Brien burned his draft card in public with the admitted intent of protesting current policies. This falls into political, core speech which is highly valued. However, his violation falls into the Universal Military Training and Service Act, the amendment specifically having continual possession of draft card. The law is supposedly to protect someone from burning someone else s draft card. The amendment is more likely to stop protest of the Vietnam War. The Court states that it s not the role of the Court to determine an illicit purpose of a law. The core argument for O Brien is that the government is going after the message, to which the government says they not after the message, just the conduct.

And because its only conduct, the O Brien test applies: 1. 2. 3. 4. It is within constitutional power of government It furthers an important or substantial government interest The government interest is unrelated to suppression of expression; and Any incidental restriction on expression is no greater than essential to further government interest

Two issues: think in terms of purpose and effect 1. Purpose: what is government aiming at? O Brien applies only if government isn t aiming at expression 2. Effect: what does the government hit? As long as government isn t aiming at expression, it is OK if government incidentally hits it What impact as to O Brien s conduct? Can he go somewhere else and burn his draft card A symbolic speech regulation completely restricts the action

Time, Place, or Manner Regulates when where and how you say something, but not what is said.

Government says it doesn t want to prevent your speech, it just wants to prevent you from speaking at a particular time or place, or in a certain way Cases allow time, place, or manner regulations so long as the government can show that it has a sufficient non-content interest and that the regulation allows alternative opportunities for the speech Focus is on low value sexually-oriented speech, and the general stance is that it is on the fringe of 1st AM protection Adult bookstores and movie theaters, and strip clubs Government is allowed to target the secondary effects of speech Bad things crime, disease, decreased property values happen if this speech is allowed

Page | 60

Time Place or Manner Test Four Elements: 1. Must advance a significant government interest (key point is that its intermediate scrutiny) 2. Must be content-neutral, meaning it must be justified without reference to the content of the speech 3. Must be narrowly tailored (Sounds like strict scrutiny, but not the same as Equal Protection or Due Process meaning. It is substantially less than those.) 4. Must leave available ample alternative channels or opportunities for information to be communicated. Meaning, if the government wants to regulate under this regulation, there needs to be an alternative. Two issues: think in terms of purpose and effect 1. Purpose: what is government aiming at? Time, place or manner applies only if government isn t aiming at the specific message the content 2. Effect: what does the government hit? As long as government isn t aiming at the message the content, it is OK if it hits how it is being said the expression itself What impact as to O Brien s conduct? Can he protest the draft in a different manner?

There is a different focus on this from O Brien. In that, it has to show that it is unrelated to the suppression of expression. Here, it has to be shown that the regulation is content-neutral. This can be shown by stating that the expression can t be done here, but can be done elsewhere.

Narrow Tailoring Not the same standard as in equal protection and due process Ward v. Rock Against Racism Narrow tailoring does not mean least restrictive means - Satisfied so long as the regulation promotes a substantial government interest that would be achieved less effectively absent the regulation. - Under time, place or manner, narrow tailoring does not mean least restrictive means o [N]arrow tailoring is satisfied so long as [the] regulation promotes a substantial government interest that would be achieved less effectively absent the regulation. - The same lower standard for narrow tailoring applies under O Brien. See Rumsfeld v. Forum for Academic and Institutional Rights, Inc. (2006)

Texas v. Johnson A Texas statute restricts desecration if it will seriously offend one or more persons. Johnson s burning the flag fit the restriction. Johnson was obviously protesting something in burning the flag, but there was nothing particular in his message. The next question is whether observers will understand what he s stating. The Court uses the following analysis:

Page | 61

Expressive Conduct Regulation Two part analysis 1. Was the conduct expressive in nature? Based on factual evidence from view or speaker and the listener y Was there an intent to convey a particularized message? AND y Was there a great likelihood that the message would be understood by those who viewed it? y If no, there is no 1st Amendment issue st y If yes, question #2 and it is a 1 Amendment issue. The next question looks at the specifics 2. Is the state s interest related to the suppression of expression (which looks at what the interest of the government is, which CANNOT be within expression) y If the government is NOT aiming at expression, apply the O Brien test y If the government is related to suppressing expression, but not the content, O Brien doesn t apply There needs to be a determination of whether it is a content-neutral time, place, or manner restriction. y If the government restriction is content based, it s now subject to strict scrutiny review.

The government is not going after Johnson s expression, but his content. Texas then points out that flag burning will lead to breaches of peace will not justify speech regulation The second interest of Texas is preserving the flag a symbol of nationhood and national unity. Brennan sees the interest to be content-based. The admission is that Texas is after the content of speech. This acknowledges that the only concern in the regulation is content (burning a flag) Dissent sees that an exception should be needed. The response is that the reason of the flag shouldn t get an exception and if it s given, where does it stop? No formal realizability. Also, dissent argues there is no expression, only conduct. The key point of the case is the model for figuring out which of the tests should be applied whether it s an O Brien or TPM exception.

Clark v. Community for Creative Non-Violence Court assumed that sleeping as part of this demonstration was expressive conduct That sleeping may be symbolic speech does not mean government cannot regulate [T]ime, place, or manner regulations normally have the purpose and direct effect of limiting expression but are nevertheless valid. Group wants to demonstrate for the homeless by sleeping overnight in Lafayette Park. Regulation in place the does not allow sleeping overnight in the park. CCNV s argument against this is that restriction is changing their message. The Court applies both O Brien and TPM test, because of the ability to view the regulation in different manners. The regulation could be a considered a complete ban on expression by barring sleeping anywhere in Lafayette Park. Use O Brien as a result. Another point is that it could be seen as a ban on expression in specifically in the park and its only a place restriction.

Page | 62

The government s argument is that the restriction is simply for keeping the park clean and safe, a legitimate interest unrelated to the suppression of expression. The question of narrow tailoring is answered by whether the interest is better off with the regulation rather than without it. The regulation passes both tests and is upheld.

Secondary effects doctrine A government states that there is no concern about content but the secondary effects resulting from the content makes a restriction allowable. Low Value Speech - Typically pornography related, and zoning restriction enforcement in question. Because of the zoning element, regulations are typically TPM related issues. Arcara v. Cloud Books The Court sees that the restriction is similar to other public health and safety measures. For example Government can promulgate generally applicable laws that apply to those engaged in protected First Amendment activities Law did not single out bookstores or other First Amendment activities for burden Not immune from fire code or other public health regulations just because First Amendment implicated

Young v. American Mini Theatres Zoning regulations on adult movie theaters. The key point in the case is that the opinion outwardly states porn to be low value speech and that a content based regulation related to porn is allowable. It places porn and other low value speech into its own category.

In a footnote, Court said that the ordinance was applied to the secondary effects of adult theaters that allow areas to deteriorate and become a focus for criminal activity.

Renton v. Playtime Theatres Zoning ordinance on location of adult theaters, pointing out that it is a TPM restriction. Even though adult theaters are mentioned, it focuses on protecting the city from crime, etc. the secondary effects of adult theatres is the supposed predominate reason for the ordinance, not a content based restriction. It looks back at Young and re-characterizes it as a result. Another issue in the measure is that the ordinance eliminates 94% of the city land available. The last TPM measure is alternate option. The Court sees this as allowable because it s not the government s job to make sure that they have something available to have their theatre

Court upheld a zoning ordinance regulating adult movie theaters But Court described the ordinance as content-neutral

Page | 63

Said the city was predominately concerned with secondary effects: preventing crime, maintaining property values, and preserving quality of neighborhoods - Recharacterized Young as secondary effects case Since ordinance was content neutral and directed to secondary effects, Court reviewed under time, place, or manner test Did the ordinance Serve a substantial governmental interest? Yes Allow reasonable alternative avenues of communication? Yes - Didn t matter that almost no land available in city - Only required a reasonable opportunity to operate theater

City of Erie v. Pap s A.M.

O Brien test applied since it was expressive conduct Law banned all public nudity, not just that associated with dancing, so law was content-neutral If states can regulate harmful secondary effects, then de minimis intrusions on expression such as those at issue here cannot render the ordinance content-based. City had important interest in public health and safety (controlling secondary effects) that was unrelated to suppression of speech Was the government incidentally banning a message? Or just regulating the manner in which the message could be conveyed? For O Brien to apply, which one does it have to be? Why not apply time, place, or manner? - If the message was conveyed by totally nude dancing, did this regulation leave adequate alternatives?

Ordinance banning dancing in a state of nudity. The Court sees that being in a state of nudity can convey a specific message, in this case erotic expression. As a result, O Brien test applied. Also, low value speech measure comes up putting it at the fringes of 1st Amendment which the Court has used in the past that allows for restriction. However, there is historical evidence that supports the idea that the 1st is in place to protect minority view speech. As expressive conduct, O Brien applies and O Connor compares to draft card burning. One message is being conveyed in the nude in an erotic manner, and an argument that the message/expression is being banned. O Brien regulation. Alternately, usage of pasties and g-string can be seen as a complete alteration of the message, but the Court sees that it is only about changing the manner only. As a TPM issue, the restriction is allowable. The Court points out that Erie is specifically concerned about naked dancers and secondary effects. Erie proves the secondary effects with nothing substantial, only anecdotal information. Erie also refers to a Seattle measure that puts all porn in one location based on a study they actually developed. As a result, proving secondary effects does not have to be substantially proven.

Page | 64

Secondary effect can apply in either O Brien or TPM regulations. As the government is interested in secondary effects, proof of the restriction reducing them is not needed. Essentially, the only question asked is whether things are better off with it than without it. Stevens dissent To believe that the mandatory addition of pasties and a g-string will have any kind of noticeable impact on secondary effects requires nothing short of a titanic surrender to the implausible.

Commercial Speech Form of intermediate scrutiny and content based speech. Government can regulate the content of commercial speech and such regulation generally receive an intermediate level of scrutiny. Until the late 60s, commercial speech had no 1st Amendment protection. The reasoning for no need is that in theory, the economic motive is what drives commercial speech and will be less likely to be chilled.

Commercial speech is expression related solely to the economic interest of the speaker and its audience. Central Hudson Fact that advertiser s interest is economic does not preclude protection of the advertiser s speech. Virginia State Bd. Of Pharmacy

Virginia State Board of Pharmacy v. Virginia Citizens Consumer Council Informative content and persuasive content present in this speech allows for some 1st AM protection, but it is not entirely excluded from regulation Ohralik v. Ohio State Bar Assn A commercial transaction that involves speech means the government could have an interest. The government has an interest in protecting consumers from fraudulent or coercive speech in solicitation for representation. Since the interest is to protect consumer, it s content neutral Central Hudson Gas & Electric v. Public Serv. Comm n Created test used to determine validity of commercial speech Central Hudson intermediate scrutiny type test controls 1. Speech must concern lawful activity and not be misleading, if it is, no protection 2. Government must have substantial interest

Page | 65

3. Regulation must directly advance gov t interest 4. Regulation cannot be more extensive than necessary to serve government interest. The substantial overbreath doctrine does not apply to commercial speech Lorillard Tobacco v. Reilly Court asked to reject Central Hudson and apply strict scrutiny, Court declined Reinforced that the Central Hudson standard holds for commercial speech Elaborates on the last two steps of the Central Hudson test. Third step requires that means selected by state directly advance state interest. Government must show, by evidence and not by speculation, that ha Fourth step says regulation cannot be more extensive than necessary to serve gov t interest. Least restrictive means reasonably fit between state interest and means chosen to achieve that interest.

Prior Restraints Certain efforts by the government to suppress speech before it happens are categorized as prior restraints, and are presumptively invalid. Thing is, everything is pretty much prior restraint Most prior restraints come in the form of: 1. An administrative license or permit scheme that forbids speech in advance of the time it is to occur (parade permits); or 2. A court order forbidding speech, or penalty of contempt Near v. Minnesota (injunction). Thing with this is that typically an injunction will be requested after the speech has occurred. Further, the injunction will be reviewed judicially. Better for the speaker because there is an opportunity to correct the behavior as opposed to a criminal statute in place With a true prior restraint, a requirement exists to obtain permission to do something prior to actually doing it. Prior restraints can be difficult to identify There is a presumption of unconstitutionality of prior restraints. Courts are reluctant to find a law or order to be a prior restraint because they are so strongly disfavored While ordinary laws that punish speech can have more chilling effect, the collateral bar rule makes prior restraints especially harsh because the rule states:

Must obey a court order until it is lifted, expires, or dissolved Cannot disobey just because you this it is unconstitutional Violation of the order also forfeits the right to challenge the underlying conclusion of the Court

Page | 66

License & Permits Court has announced three requirements for license or permit scheme to be valid First, the government must have an important reason for the license or permit requirement From Cox v. New Hampshire, Adequate security need met by telling when the parade will occur and Also that there would not be conflicting parades

Second, the standards for granting or denying picense or permit must be clear and must afford almost no discretion to government Kunz v. New York Ordinance prohibiting religious meeting on public street without a permit was invalid because it gave an administrative official virtually unlimited discretion to decide what was a religious meeting. Too much discretion because of ability to define a religious meeting Third, must be procedural safeguards in place tp assure prompt decision and judicial review

Must be prompt decision Must be full and fair hearing There will also be an appeal/hearing function to the adverse determination. It will be adversarial, and evidence show why it should be denied or allowed Must be prompt and final judicial determination

Freedman case outlines the standard Injunctions Near v. Minnesota This is the essence of censorship Proper remedy for false accusations is subsequent under

National Security New York Times v. United States (Pentagon Papers) Prior restraint denied even in a situation where the executive branch warned of danger to national security Justice White takes a Youngstown approach stating a lack of authority Obscenity Unprotected speech, no limit on local authority to regulate potentially obscene material Times Film Corp v. Chicago

Page | 67

The Court is reluctant to prior restraint Alexander v. United States $9 million in inventory destroyed when only 7 obscene items found The Court said no prior restraint even though it essentially was

Freedom of the Press: Newsgathering While freedom of the press is mention separately from speech and other rights, much free press doctrine is covered by free speech doctrine. As a general rule, the press does not receive special legal privileges or protections simply by virtue of being the press Two categories: Defensive use of press clause and offensive use of press clause The Court has pretty much said the press cannot use the 1st as a shield that is to resist government process nor can the press use the 1st as a sword to gain access to things the general public doesn t have access to.

Defensive use Press clause does not shield the press from government process subpoena and testimony. Bratzenburg y State law may grant privilege but there is no federal reporter shield law. y Press clause does not immunize press from search warrants. Zurcher Offensive Use Press clause does not assure any greater access than general public. Richmond Newspapers y There is a 1st Amendment right that trials be open to the public General Laws - Press not exempt from generally applicable laws - But taxes directed particularly at press are unconstitutional (any special burden at the press may not be constitutional the following were not upheld o Grosjean license tax on ads in publications o Minneapolis Star tax on newsprint and printers ink

Branzburg v. Hayes (protecting confidentiality of sources)


Court said great weight of authority declined to recognize exemption from the press Unless privileged, the public has a right to every person s evidence Court declined to create 1st Amendment privilege Justice Powell 5th vote would engage in a case by case balancing to determine whether privilege exists (most courts will do this balancing) The Court points out the press shouldn t be exempt because defining the press would be difficult to establish.

Page | 68

Also, there is a lack of formal realizability. If there is to be a special exemption for press, legislature is who makes it happen.

Zurcher v. Stanford Daily (search warrant issue) - No special 1st Am press exemption from search warrants - Court said normal requirements applicable to search warrants probable cause, particularly as to places to be searched and things to be seized, and reasonableness were sufficient protection - Justice Powell again took a more middle ground approach, seeing that a balancing is required - Note that post-Zurcher, Congress stepped in - Privacy Protection Act of 1980 - Offered some protection to press from searches - Generally, searches of those reasonably believed to be disseminating news to the public prohibited unless: o Probable cause or o Notice by subpoena would cause loss of evidence Offensive Use Court has never held that press gets special access because of press status Richmond Newspapers v. Virginia - 6th Amendment applies to the accused, as to the general public and press, there is nothing textual to gives them the right to public trial. - Trials being open affairs have been something always available, and is enforced by the 1st Amendment. Also, the freedom of speech includes the right to associate with the same people and listen. - Justice Stewart points out that in times with limited access, the press might get some preference because they can share their observances with the general public. - Right of access not absolute If court proceedings are to be closed, there needs to be - Compelling gov t interest - Narrow tailoring - Case by cases, no blanket closure rule - Specific findings to justify closure - Waller v. Georgia elaborated this closure o Party seeking closure bears burden Globe Newspaper Co. v. Superior Court (1982) Involved law providing that trial courts could close trial to press during testimony of underage victims of sex crimes Closing court proceedings allowed only if necessitated by a compelling governmental interest, and is narrowly tailored to serve that interest. Law failed narrow tailoring because blanket closure rule inappropriate; should be case-by-case basis

Page | 69

Speech Forums The extent of protection for speech on public property may depend on the nature of the forum, or place, where once speaks. The Court has identified these types of forums: public, unlimited designated public, limited designated public, and nonpublic. The forum analysis does not apply to private property. Categories of speech forums 1. Traditional Public Places that have historically and traditionally been open by the government for speech activities. There is no list for this however. Ex. public roads, sidewalks, courthouse steps, town square.

Government has an affirmative duty to keep these open Any restrictions must be content neutral, if not, strict scrutiny required TPM allowed, based on the fact that they are content neutral

2. Non Public Forum military bases, jails, prisons, state run mental institutions. Places where there has never been a government enforced right. Speech regulations must be Reasonable Viewpoint neutral These regulations can in part be content based. The government can exclude speakers and subjects that it does not want spoken.

Intermediate Category the government is not required to keep them open, they can close at any time. It could be that the same location be public and non-public. Ex. only certain hours where designated forum 3. Designated Forum The median between traditional and non public. The government s actions must be voluntary and affirmative steps to open up a non public forum. Non-public forum, such as a public school, if the school board adopts a policy to allow people to use the facilities after hours and weekends for speech and expressive activities. The government has voluntarily taken steps to make a non public forum into a designated forum. a. Unlimited Designated public forums open to any material Limitations the same as public forums b. Limited Designated public forums only for specific material. Ex. Community service groups only can use the school on the off hours. Limitations will be the same as a non public forum.

Page | 70

Classification System Leading case classifying and describing types of forums is Perry Educational Ass n. v. Perry Local Educators Ass n. (1983)

Public forums are those places which by tradition have been devoted to speech and assembly Streets and parks are quintessential examples Content-based regulations here subject to strict scrutiny Designated public forum is property that government affirmatively and voluntarily opens up for expressive activity No obligation to keep open indefinitely Depends on whether limited or unlimited Nonpublic forum is any property not a public or designated public forum Must be reasonable and viewpoint neutral

Schneider v. New Jersey (1939) City has other means to prevent littering, like punishing those who litter Right to use public forums isn t unlimited State can regulate so long as regulation does not abridge constitutional right to speak Speaker cannot exercise right by, for example, standing in middle of street and impeding flow of traffic - Prohibition of such conduct allowed since it is not necessary to the right to speak Cox v. New Hampshire (1941) Government may require a permit or a license to use a public forum for parade or demonstration May also charge reasonable fee incident to permitting or licensing scheme. But cf. Forsyth County v. The Nationalist Movement (1992) (speech cannot be impaired by imposing fees and costs) Heffron v. Int l. Soc. for Krishna Consciousness (1981) upheld rule barring distribution of printed materials and solicitation of funds at county fair as reasonable time, place, or manner restriction Chicago Police v. Mosley No picketing unless labor disputes in connection with school matters Content based regulation. The Court analyzed as an equal protection matter and saw the ordinance bans speech. Strict scrutiny applies to this because it is a public forum (on the street)

Creation of new forums Int l Soc for Krishna v. Lee

Airports (terminal specifically) not traditionally a public forum no history of airports being used for speech activities no long standing history and tradition of holding airports open for public forum. Because of that, the regulation is reasonable and viewpoint neutral. Whether a speech is reasonable is found out by the intended or dedicated use of the property. If it is, good to go. From the, the airport argues that it was never a place meant for public forum and it s reasonable to prevent passengers from being solicited while they go to their flights. This case illustrates how difficult it is to move from a traditional forum to a non-public forum. It also shows what kinds of speech regulations are reasonable.

Page | 71

The Court states is that it isn t willing to expand the traditional public forum

Lee v. ISKCON deals with ban on distribution of literature, the other deals with solicitation. Here, O Connor states that a complete ban of literature distribution is not reasonable. It doesn t interfere with the operation of the facility. It illustrates reasonable v. unreasonable regulations.

Designated Public Forums A place that government voluntarily and affirmatively opens to public for speech Fact that speech activities might take place does not mean that government has voluntarily and affirmatively opened r In other words, no such thing as a designated public forum by adverse possession r Standard depends on whether it is a limited or an unlimited designated public forum Widmar v. Vincent (1981) University made classrooms and buildings available but barred student religious group Court said religious group could not be excluded Good News Club v. Milford Central School (2001) School district allowed school buildings to be used by various community groups Refused to allow Good News Club to use for religious activities Court said this was illegal viewpoint discrimination in a limited designated public forum

Non-Public Forums Any place that is not a public forum or a designated public forum is a non-public forum Cornelius v. NAACP Legal Defense Educ. Fund List of designated charities that provided funding excluded NAACP. Court ruled that combined federal campaign was a non-public forum and they had the right to exclude as long as it was reasonable and viewpoint neutral. Because the list was mostly children s hospitals, etc., a legal fund wouldn t be included. Parade example group wants to be in parade but excluded Illustrates how a list can be an intangible forum

Speech Forums Government as Speaker When the government acts as educator, government interests may justify more extensive regulation of speech Students have free speech rights. Tinker Schools may regulate student speech when It is profane, indecent, or sexually explicit. Bethel It disrupts school activities. Tinker It is associated with certain school-sponsored activities. Hazelwood

Page | 72

Conditional spending similar in speech subsidy cases the Court leans on the choice of turning down funding but there is nothing in place that keeps you from stopping what you re doing, it s just that the government isn t going to pay for it.

The government is allowed to have its own message except for religion and it can express that message. It imposes no constitutional requirement to present an opposing viewpoint In other situations where government acts as educator or speaker, it has had varying success regulating the content of speech Government may make content-based distinctions when acting as educator Government may subsidize private speech, but need not subsidize all of it Government as speaker is allowed to communicate its own message

Rust v. Sullivan

Title X restricts federal funding to health care if abortion is used, discussed, or referred. Content based restriction. The Court does not see a content based, viewpoint restriction, only something that the government is funding one activity and not another. The Court also does not see this as a punishment, only that the government is going to pay for one type of speech, but not another. The government has a message it wants to convey family planning and childbirth good. And the government is willing to pay for that message to be passed on. Family planning grant recipients are barred from using grant funds to advise or counsel regarding abortion Government may fund activities it deems desirable without being obliged to fund other activities it deems undesirable No viewpoint discrimination; merely funding one activity but not others Funding restriction is not a restriction on speakers, it is a restriction on activities Clinic workers and employees remain free to speak about abortion, just not in connection with grant project Dissent argues that restriction is a viewpoint discriminatory speech regulation

National Endowment for the Arts v. Finley (1998)


Conditional spending four factor brought down to a single factor is withholding the money depriving a constitutional right? If not, its constitutional Content-based considerations are the nature of arts funding Limited funds and most applicants are rejected Neutrality is impossible and NEA is required to make aesthetic judgments [U]ntil [the statute] is applied in a manner that raises concern about the suppression of disfavored viewpoints, we uphold [its] constitutionality . Court also noted that government has wider latitude in matters of spending Government may fund based on criteria that would be impermissible if government were trying to directly regulate Congress has broad latitude to set spending priorities so long as the law does not infringe on other constitutional rights. May fund one activity without funding others

Page | 73

Government as Educator School Speech Model 1. Tinker v. Des Moines School District kids wearing armbands in protest. The Court held that a restriction for this is not valid, students have 1st Amendment rights in public schools, are not coextensive with 1st Amendment rights outside the school setting. Tinker Test: As long as the activity doesn t disrupt class, school activities and doesn t harm the rights of other persons at the school, it is allowed.

Dissent sees that the school should be making the rules. The trend since this case has become much more restrictive.

2. Bethel speech by kid at assembly to support friend running for student government. Bethel Test: If speech was sexually suggestive, patently offensive, lewd, or indecent the school can regulate it.

3. Hazelwood v. Kuhlmeier school principal has the final say in the paper s content. He rejects two pages for content about teen pregnancy. Content based outside the school, it s a problem. However, the Court holds that the school paper is not a public forum, it s a school project to learn about journalism.

This is a question of when a school must affirmatively support a certain kind of speech. Hazelwood Standard: Educators may exercise editorial control over content of school sponsored expressive activities so long as reasonably related to legitimate pedagogical concerns. Close to rational basis review. Very permissive standard for content based regulation United States v. Amer. Lib. Ass n. (2003) Federal grant program placed conditions on receipt of federal funds Required Internet filters to prevent children from viewing adult content and obscenity Public forum analysis and heightened judicial scrutiny are incompatible with discretion public libraries must have to fulfill their traditional mission Library s decision to exclude pornography from print collections not subject to strict scrutiny, so no reason to subject Internet filtering to strict scrutiny here Public library staffs necessarily consider content in making collection decisions and enjoy broad discretion in making them.

Page | 74

Adults are not denied access to non-obscene, pornographic materials Analyzed as a spending measure, law did not violate patrons First Amendment rights and did not induce libraries to violate Constitution No public or designated public forum No history of public library Internet access as a public forum Library did not create designated public forum because purpose of Internet access in public library is not to create a speech forum but to facilitate research, learning, and recreation Justice Souter s dissent distinguished acquisition from censoring or removal

Regulation of Media Traditional broadcast media typically at issue in these matters. Satellite, cable, DSL don t use the same public access media venues. However, the standard to use for those media is in debate. Profane or indecent speech can be regulated by the government. The level of regulation will depend on context (8pm v. 2am) and as such the government s interests will vary, a sliding scale depending on the context. Pacifica Cable system regulation not as clear but content-based regulation subject to strict scrutiny. Playboy Entertainment Group. The issue with these cases has been the attempts to regulate away from children also keeps it away from adults, which isn t acceptable.

Adult Interest content Efforts to criminalize dissemination of non-obscene adult material when viewed by minors have failed. Reno But Congress can condition receipt of federal money for providing Internet access in public libraries on use of adult content filters.

FCC v. Pacifica Foundation Not protected in this context for two reasons 1. Uniquely pervasive nature of broadcast media y In the home, right to be left alone wins over First Amendment right of speaker 2. Broadcasting is accessible to children y Government has interest in protecting children Note that Pacifica did not apply strict scrutiny to this content-based regulation Based decision on characteristics of broadcast medium These characteristics justified government regulation

United States v. Playboy Entertainment y Content based restriction on Playboy Entertainment by limiting broadcasting to 10pm to 6am only. y Broadcasting blackout of 16 hours has been created, which is not the least restrictive means under strict scrutiny review.

Page | 75

y y

House to house blocking is more appropriate, addressing the compelling interest while being the least restrictive means at the same time. The case is significant because it deals with non-traditional media

Reno v. ACLU Internet act, the Communications Decency Act criminalized transmitting obscene material, struck down because of vagueness and overbroad. Court said law was vague and overbroad Neither indecent nor patently offensive were defined Vagueness likely to have a chilling effect It also incorporated the community standards from Miller, but the problem with that is that it would have to use the most sensitive community because of the nature of the internet. The COPA measure was Congress response.

Ashcroft v. ACLU Child Online Protection Act was the response to Reno v. ACLU Revised statute was limited to patently offensive speech Appearing on the Web Made for commercial purposes Appealing to children s prurient interest; and Without artistic, literary, political, or scientific value for children not judged by community standards Court upheld injunction against enforcement of COPA As in Playboy Entertainment Group, statute failed less restrictive alternatives test Blocking and filtering software would be less restrictive and more effective than COPA ban Government bears burden of proof and failed to show blocking and filtering would be less effective than COPA restrictions

Freedom of Association Issues can be put into three categories 1. 2. 3. Freedom not to speak or support ideas Freedom of association in employment Impact of anti-discrimination of freedom to associate

Rules discussed here do not impact intimate associations. The kind of association is expressive association derives from the 1st Amendment and protection of speech. Not the same as intimate association, which is protected under the 14th Amendment (fundamental right to live with family)

Page | 76

Cannot be compelled to believe in ideas or concepts found to be offensive. Government can t do that. Wooley v. Maynard, NH live free or die on license plate and person didn t want to have it on his car. The Court held that he didn t have to show it if he didn t want to because it was an idea that was offensive to him. Johanns v. Livestock Congress mandated campaign for beef promotion, some producers did not agree with the message and did not want to be support the campaign. The Court held that the government can compel certain kinds of speech. Ex. Filing and signing tax return, forcing attending a grand jury. The general rule is that government can t compel speech, but there are exceptions. Hurley v. Irish-American Gay Lesbian Group St. Patrick s Day parade private group organizer wouldn t allow a gay group to be in their parade. Under this case, a private case had the right to choose its message can bar certain messages if it chooses to.

Boy Scouts v. Dale Dale, a gay employee for the Boy Scouts got fired when he came out. The Act barring discrimination was held invalid in this situation because it impinged on the Boy Scouts right to state its message of being against homosexuality. The Court points out that it will defer to the speaker s message and opinion on what might infringe on what its message is. Once stated, courts will defer to it. However, that is challenged in the Rumsfeld case.

Rumsfeld v. FAIR Held that law schools couldn t bar military recruiters, if they did funding would be eliminated. Forcing the sending of emails was FAIR s argument, but the Court held that it wasn t expressive nature, just conduct. Also, FAIR argued that it was a forced association because it forced the schools to allow recruiters on campus. The Court held that it wasn t forced association; the recruiters were not becoming a part of the schools. This ruling was in challenge to the Dale decision. 1. Private groups can choose 2. They can prohibit messages that it doesn t want expressed. Employment & Commercial Interest Connick v. Myers Covers government employees and the restrictions that government can place on them. Government official may fire employees for on the job speech that cannot be fairly considered as relating to any matter of political, social, or other concern to the community The public concern doctrine is created - Whether an employee s speech deals with a matter of public concern is determined by the content, form, and context of a given statement

Page | 77

Matters of public concern receive heighten scrutiny; matters of personal interest do not.

The Court holds that the questionnaire was related to personal interests, and as such is not protected speech, as opposed to a matter of public concern. From the case, the basic dividing line is public concern v. private interest. If speaking on matters of private interest, there is no 1st Amendment protection. If speaking as a citizen as a member of public concern, they may get 1st Amendment protection.

Pre-Garcetti Two approaches: 1. 2. Private Concern No 1st Am Protection Citizen-public concern Pickering balancing applied, and protection based on the balance outcome. (adding from Garcetti) within scope of duties, even if public concern No 1st Amendment protection

Pickering Balancing Employee and public interest in speech v. employer interest in running an effective and efficient office.

Garcetti v. Ceballos Employment issue again. DA writes a memo in his duty to dismiss case because of a bad search warrant what had false representation, which is a public concern. Afterward, he s transferred and denied a promotion. The Court holds that the memo was written within the scope of duty, and not speaking a citizen, but as an employee and as such no protection under the 1st is available. Revisiting the pre-Garcetti, the Citizen-public concern is changed Pickering balancing applied, and protection based on the balance outcome. Citizen-public concern (adding from Garcetti) within scope of duties, even if public concern No 1st Amendment protection

Freedom of Religion: Establishment Clause As with speech, all kind of laws dealt with religion. Historically, the Establishment Clause applied to the federal government only. Its intent was to prevent the federal government from establishing religion and also preventing the fed gov t from forcing religion on the states.

Page | 78

The 14th Amendment changed things; Everson v. Bd of Ed. held that the 14th incorporated the Establishment Clause into the States. Everson held that a bus transportation to public and private school was a general neutral benefit. Neutral laws are key in establishment clause doctrine. Establishment Clause Tests (use all three test for an analysis) Lemon Test after Lemon v. Kurtz. Had three parts and all must be satisfied. 1. Purpose Test - The law or regulation must have a legitimate, non-sham, secular purpose. Which states that there may have had more than one purpose. As long as at least one of them is legitimate and secular, it will be fine. (easily satisfied, as long facially neutral) 2. Effects Test The principal or primary effect cannot be to advance or inhibit religion in general or one religious sect over another. If the effect is religiously neutral, it will pass. (Key area of analysis) 3. Excessive Entanglement (now part of Question 2 because of Aguilar) The law cannot result in excessive entanglement between government and religion. Entanglement is what is looked at to determine whether a law is neutral. Points on Lemon - Secular Purpose o Sometimes called purposive neutrality o Typically not a difficult requirement for government to meet. o Mueller v. Allen Court held essentially that aiding parents with getting their kids into school is facially neutral, even if the parents elect to send their children to non-public schools. - Effects Inquiry o Most analysis from school funding and religious speech  In both, the key issue is neutrality, and will be neutral if: y No reference to religion in defined benefit recipients y No governmental indoctrination  In religious speech, no violation if forum is opened and allow equal basis to both religious and non-religious representation. o In both these issues, the key factor is whether or not the government is providing something in a neutral way. Endorsement Test Traced to O Connor s concurrence in Lynch v. Donnelly. The intent was to only be a modification of the Lemon, but it established its own distinction. 1. Purpose prong - Speech and act is looked at from speaker and viewer perspective. And it is asked whether the actual purpose is to advance religion. (similar to neutrality in Lemon)

Page | 79

What does it take to reflect a purpose to advance religion? Undefined, but will look at history, floor statements, legislative reports, etc. 2. Reasonable observer standard - Whether a reasonable observer would understand that the purpose is to advance or endorse religion. A reasonable observer is someone familiar with history and context. If both are met, endorsement established. The reason for concern of endorsement is that if the government endorses a practice, insiders are created for those who are a part of the religion endorsed, and outsiders are created to those who aren t a member of that religion. Coercion Test Majority view (Kennedy in Lee v. Weisman) Coercion brought out peer pressure coming as a result of a measure or law. The less oppressive is sufficient, and may depend on context. Kennedy s version is more fact and circumstance oriented than the minority view. Psychological coercion is sufficient to establish coercion, from Kennedy s opinion from the high school case Minority view (Scalia and Thomas) no coercion unless threat of imprisonment for not following. It has never been the view of the whole Court. Pretty much it means coercion never happens No elements, but views are available Today, the Court does not favor one or the other. There has been distain of the Lemon test, but nothing better has come up. O Connor was the key on the endorsement test, but with her gone it might not last. The coercion test could take the lead with Kennedy being the median vote.

Aid to Religion Government giving funds for education materials or in kind donations, voucher programs, tax deduction Two Questions for Aid to Religion issue: 1. Private Choice Doctrine Is the government aid being supplied directly to religious schools by the government, or indirectly as a result of private choice (parents choosing school on their own)? a. State allocates funds based on where the parents choose to send their children b. If the private choice is at hand, it severs the causal indoctrination chain, and not attributable to anything the government has done c. Examples Witters state ed grant to blind student, who chose to attend private religious college 2. Indoctrination Question If aid goes to religious institutions other than through private choice, if the aid goes directly to the schools, the question asked is whether the government has engaged in

Page | 80

religious indoctrination as a result. There is no presumption on this, the challenger has to prove the aid resulted in religious indoctrination. (check slide on this) Religious Speech Need to distinguish between private versus public speech y Private speaker is entitled to voice religious or irreligious preferences Establishment Clause is not concerned with this because it is private action. y The concern is whether the government specifically has engaged in religious speech, or attribute to the government private religious speech. Private speech in public forums y When government merely permits, on a nondiscriminatory basis, private religious speech in a public or designated public forum, no Establish Clause violation y Widmar v. Vincent no Establishment Clause issue for making available a public forum because it was open to all, on an equal basis (see slides) Symbolic Speech More difficult to determine when symbols are on government property - Issue is whether it s private or public speech - Endorsement test seemed to be preferred analytical model until McCreary (purposive neutrality) and Van Orden (history)

Lynch v. Donnelly Applied Lemon test in concluding that display of city-owned crche, as part of a holiday display, was not an Establishment Clause violation. - No effect of advancing or promoting religion - No more an endorsement than others Court had allowed in the past. The key point is that the crche was blended in with other non-secular symbols, and the overall message is no longer religious Allegheny County v. ALCU Creche display inside county courthouse and the only thing present, nothing non-secular along with it. - And in using Endorsement test, it was a religious message. - Essential principle of the Est. Clause was that it prohibits government from appearing to take a position on questions of religious belief or from makin adherence to a religion Majority - Favored endorsement - Context mattered Dissenters - Pointed to history of religion in public sphere and downplayed religious significance of displays - Would apply much more narrow test like coercion Van Orden v. Perry Historical analysis was used. It looked at how historically there have been religious holidays and traditions observed, example of Thanksgiving, State Chaplin, etc. - Note: Jefferson opposed a national Thanksgiving as he saw it was an Establishment Clause violation

Page | 81

The problem is that a historical analysis allows for selective points on either side. Lemon test not useful in assessing the passive monument state erected on public grounds. Acknowledged that Ten Commandment a religious symbol, but said OK because of history of religion in public affairs. Historical analysis was what allowed the monument to stand.

McCreary County v. ACLU - Poster of the Ten Commandments in courthouse - Focus was on purposive neutrality. - The Court found on this ground that McCreary was not acting in a genuine secular purpose, but a sham secular purpose. Symbols (In God We Trust on money, etc.) have also come with support through ceremonial deism meaning that the symbols have been used so much that they have lost specific religious message.

Religion in Public Schools y A much more bright line at school and allowing religion in the public schools. Lemon test and coercion used in analysis. y Key difference in religious private speech versus religious public speech; private is NOT touched. Prayer cases Lemon or coercion analysis - Wallace legislative history showed that the only purpose was to return prayer to public schools - Lee applied coercion analysis to strike prayer at school graduation ceremony Symbolic Speech - Stone v. Graham State law required posting of Ten Commandments in public school classrooms - Court found that the law forced children to venerate the Commandments. Curriculum Cases - Involve efforts to introduce creationism (or creation science or intelligent design ) into science classroom - Court has repeatedly rejected; the following cases were turned down: o Epperson v. Arkansas statute prohibiting evolution o Edwards v. Aguillard statutes requiring evolution and creation science failed under Lemon test

Free Exercise Clause Bars federal government and the states from making any law prohibiting the free exercise of religion
y y y

The literal text no law does not mean no law. Similar to Establishment Clause themes, but adds religious accommodation, and the question is how far can government go to accommodate, but not an establishment of religion Focus is on analyzing today

Page | 82

Generally two Tests Which test is applied based on what the law says Lukumi test strict scrutiny y If a law targets a religious exercise, government must schow a compelling interes and least restrictive means tailoring Smith test rational basis y If law is religiously neutral and generally applicable, government is free to regulate, even if free exercise is impaired. Free Exercise analysis needs to start with Smith, if the law is neutral and generally applicable. If it targets a religious exercise, strict scrutiny will apply. Religious Freedom Restoration Act unconstitutional as applied to states, but is applied to federal government y Created because of Congress not liking Smith rational basis application and wanted to restore Sherbert and Yoder, meaning that strict scrutiny should apply. y City of Boerne v. Flores struck RFRA as applied to states but not federal y Strict scrutiny applies to federal law Religious Land Use and Institutionalized Persons Act If state passes law the impairs free exercise, strict scrutiny will apply. y Upheld in Cutter y Required strict scrutiny for state laws. Sherbert, Hobbie, Yoder cases These cases point out strict scrutiny applies in free exercise cases. In Sherbert, unemployment compensation case, employee fired for wanting day of for Sabbath and denied benefits. - Court found no compelling interest to justify state s substantial impairment of free exercise rights - Strict scrutiny applied In Yoder, parents refused to keep children in public school based on religious belief. State truancy laws barred it. y The Court used strict scrutiny and held the truancy law was invalid. General rule, for impairments of Free Exercise pre-Smith, strict scrutiny Employment Division v. Smith (peyote case) y Anti-drug laws and drug counselors were fired for using peyote as part of religious belief. The law has no exception to the ban, even for religious purpose usage. Unemployment denied for violating the law. y Scalia stated no free exercise violation because the drug laws are religiously neutral and generally applicable. Nothing was stated about religion, and as laws like this generally will be subject to rational basis review.

Page | 83

y y y

Historically, the only time strict scrutiny review to religious impairments is if the law also interferes with another constitutional right. Yoder. Scalia stated, was a situation that it interfered with parent s right AND free exercise. The implication is that if you don t like the law, change it. Works for a majority religion, but not a minority one.

Smith Test General rule, free exercise laws that are religiously neutral and generally applicable will be subject to rational basis review. Exceptions 1. Unemployment issues 2. If a state law impair religious exercise AND another fundamental right, strict scrutiny will apply. Lukumi v. Hialeah (Santeria case) y Ordinance banning animal sacrifice, but had exceptions that included kosher killing y The Court held that the law was not generally neutral and applicable, only specific to Santeria religion and had exceptions for other faiths. As a result, strict scrutiny review applied. y Once a challenger established an impairment and not a neutral and applicable law, the burden shifts to the government to pass strict scrutiny. Conflict Between Clauses Addresses State or federal law that accommodates religious exercise that makes it easier to engage in religious practices. If an accommodation law accommodates all religion in general, there is not Establishment Clause issue, and the law will hold. Corp of Presiding Bishop v. Amos If an accommodation law targets one or more particular faiths and accommodates to the exclusion of all others, strict scrutiny will apply and will probably not hold.

Corp v Amos (see slides) Title VII restricts discriminatory hiring based on religion, with an exception to targets no particular religion. The issue is whether a non-religious teaching employee was subject to this exception. Kiryas Joel v. Grumet Belief that their children could not attend schools unless everyone was of the same faith. A special district was created as a result. The Court held that the creation of the special district targeted a particular faith. It was an accommodation targeting one religious faith, and as a result strict scrutiny will apply. General Rule: If accommodation applies to all, rational basis. If targets one or more groups, strict scrutiny is the standard.

Page | 84

You might also like